Sie sind auf Seite 1von 75

INSIGHTSIAS PRELIMS TEST SERIES 2019 – HISTORY QUESTIONS

Table of Contents MODERN INDIAN HISTORY .............................. 17


ANCIENT INDIAN HISTORY ................................. 2
BRITISH EXPANSION AND CONSOLIDATION 17
PRE-HISTORY .................................................. 2
OTHERS .................................................... 18
PROTO HISTORY ............................................. 3
THE REVOLT OF 1857 ................................... 18
VEDIC PERIODS............................................... 4
RELIGIOUS AND SOCIAL REFORM
SHAMAN TRADITIONS .................................... 5 MOVEMENTS ............................................... 20
JAINISM ...................................................... 5 THE STRUGGLE BEGINS ................................ 23
BUDDHISM ................................................. 5 PRE-CONGRESS ........................................ 23
OTHER SHAMAN TRADITIONS .................... 6 INDIAN NATIONAL CONGRESS AIMS AND
TERRITORIAL STATES ...................................... 7 OBJECTIVES .............................................. 23

MAHAJANAPADAS...................................... 7 NATIONAL MOVEMENT 1905-1918 ............. 24

AGE OF MAURYAS .......................................... 8 MITITANT NATIONALISM ......................... 24

POST MAURYANS ........................................... 8 SWADESHI AND BOYCOTT ....................... 24

SOUTHERN KINGDOMS .................................. 8 EXTREMIST LEADERSHIP .......................... 25

SATAVAHANAS ........................................... 8 WORLD WAR 1 and Indian response ....... 27

SANGAM LITERATURE ................................ 9 BRITISH RESPONSE ................................... 27

GUPTA PERIOD ............................................... 9 NATIONAL MOVEMENT 1919-1939 ............. 28

POST GUPTAS ............................................... 10 EMERGENCE OF GANDHI ......................... 28

HARSHA AND HIS TIMES .......................... 10 NON-COOPERATION MOVEMENT ........... 30

NEW DECCAN STATES .................................. 10 ANTI SIMON MOVEMENT ........................ 31

PALLAVAS ................................................. 10 NEHRU REPORT........................................ 31

CHALUKYAS .............................................. 10 CIVIL DISOBEDIENCE MOVEMENT ........... 32

INDIAN PHILOSOPHY .................................... 10 ROUND TABLE CONFERENCE ................... 33

TRAVELLERS AND OTHER SOCIAL LIFE ......... 12 OTHER EVENTS AND MISC ....................... 33

MEDIEVAL INDIAN HISTORY ............................. 14 NATIONAL MOVEMENT 1939-INDEPENDENCE


..................................................................... 35
BHAKTI MOVEMENT .................................... 14
WORLD WAR 2 ......................................... 35
SOCIAL LIFE AND DEVELOPMENT................. 14
AUGUST OFFER ........................................ 36
VIJAYANAGARA EMPIRE ............................... 14
QUIT INDIA MOVEMENT .......................... 36
STRUGLE FOR EMPIRE IN NORTH INDIA ...... 15
OTHER EVENTS......................................... 36
CONSOLIDATION OF MUGHALS ................... 15
CABINET MISSION .................................... 37
AKBAR ...................................................... 16
TOWARDS INDEPENDENCE AND PARTITION
LIFE AND DEVELOPMENT ............................. 16
................................................................. 37

WWW.INSIGHTSIAS.COM WWW.INSIGHTSONINDIA.COM
INSIGHTSIAS PRELIMS TEST SERIES 2019 – HISTORY QUESTIONS

ADMINISTRATIVE CHANGES......................... 39
CONSTITIONAL CHANGES......................... 39
GOVERNOR GENERALS AND VICEROYS .... 40
ECONOMIC IMPACT ..................................... 41
DEVELOPMENT OF PRESS ............................ 45
DEVELOPMENT OF EDUCATION ................... 46
ADMINISTRATIVE POLICY ............................. 47
THE INDIAN STATES...................................... 50
CIVIL REBELLION ........................................... 51
CASTE MOVEMENTS ................................ 51
TRIBAL REVOLT ......................................... 51
PEASANT MOVEMENTS............................ 52
WORKING CLASS MOVEMENT ................. 54
Other Movements .................................... 54
PERSONALITIES............................................. 55
MISC ................................................................. 59
Post-Independence .......................................... 62
World History ................................................... 65
ART AND CULTURE ........................................... 66
LITERATURE .................................................. 66
SCULPTURES ................................................. 67
ARCHITECTURE ............................................. 67
Music ............................................................ 69
Paintings ....................................................... 69
Dance ........................................................... 70
Martial art forms .......................................... 71
Festivals and Fairs ........................................ 71
Institution..................................................... 72
ANCIENT INDIAN HISTORY
MISC ............................................................. 72 PRE-HISTORY
Q 1. What is the most appropriate
period you would choose for the
beginnings of agriculture in the
Indian subcontinent?

WWW.INSIGHTSIAS.COM WWW.INSIGHTSONINDIA.COM
INSIGHTSIAS PRELIMS TEST SERIES 2019 – HISTORY QUESTIONS

a) 3000 BCE a) 1, 2 and 3


b) 5000 BCE b) 2 and 3 only
c) 9000 BCE c) 1 only
d) 15000 BCE d) 1 and 3 only

Q 2. Which of these regions is/are Q 4. Which of the following


well known for hosting pre-historic regions, as a part of the Harappan
rock art paintings? civilization, is associated with the
Ganeshwar-Jodhpura culture?
1. Vindhya ranges of Madhya
Pradesh a) Khetri area
2. Kaimurean extensions of Uttar b) Chottanagpur belt
Pradesh
c) Malwa Plateau in Madhya Pradesh
Which of the above is/are correct?
d) Rajmahal Hills
a) 1 only
b) 2 only
Q 5. With reference to the
c) Both 1 and 2 Harappan civilization, consider the
following statements.
d) None
1. Harappan seals were made either from
metal or clay and always contain animal
PROTO HISTORY motifs.
Q 3. With reference to the food 2. Drainage systems were only found
and related habits of the Harappan in three of the largest Harappan cities.
culture, consider the following
statements. Select the correct answer using the codes
below.
1. They consumed both millets and
whole grain cereals. a) 1 only

2. There is no evidence of b) 2 only


consumption of marine products by the c) Both 1 and 2
Harappans.
d) None of the above
3. Cattle, sheep and pig were
domesticated.
Select the correct answer using the codes Q 6. The ‘Mature’ Harappan
below. culture is dated to

WWW.INSIGHTSIAS.COM WWW.INSIGHTSONINDIA.COM
INSIGHTSIAS PRELIMS TEST SERIES 2019 – HISTORY QUESTIONS

a) Circa 1900 and 1000 BCE 1. Jammu


b) Circa 2600 and 1900 BCE 2. Baluchistan
c) Circa 5000 and 4000 BCE 3. Gujarat
d) Circa 6000 and 5500 BCE 4. Afghanistan
Select the correct answer using the codes
Q 7. The easternmost Harappan below.
site amongst the following is a) 2 and 3 only
a) Rakhigarhi
b) 1, 2, 3 and 4
b) Sutkagendor
c) 1 and 4 only
c) Dholavira
d) 1, 2 and 3 only
d) Ganweriwala

Q 10. Consider the following


Q 8. The Harappans procured statements with reference to the
materials for craft production from Indus Valley civilization time
various sites. Match the following periods.
sites with the availability of the
material/stone in that site. 1. The Ahar culture which flourished in
Kerala of present day India was famous for
1. Lothal: Carnelian its blade tool industry.
2. Shortughai: Lapis Lazuli 2. Navdatoli near Narmada River was an
3. Khetri: Copper important settlement point for Malwa
culture in Central India.
Select the correct matches using the codes
below. Which of the above is/are correct?

a) 1 and 2 only a) 1 only

b) 1, 2 and 3 b) 2 only

c) 2 and 3 only c) Both 1 and 2

d) 1 and 3 only d) None

Q 9. Weights, stone blades and VEDIC PERIODS


baked bricks belonging to the Q 11. Consider the following
Harappan Culture can be found in statements about Aranyakas.
which of the following present day
regions?

WWW.INSIGHTSIAS.COM WWW.INSIGHTSONINDIA.COM
INSIGHTSIAS PRELIMS TEST SERIES 2019 – HISTORY QUESTIONS

1. They are the concluding portions d) None


of the several Brahmanas.

2. They mainly deal with sacrificial


BUDDHISM
techniques and karma kandas.
Q 13. Which of these sutras is NOT
3. These were works to be read in the related to Buddhism?
villages, as opposed to ‘Brahmanas’ text
a) Lotus Sutra
which must be read in the forests.
b) Heart Sutra
4. There is no Aranyaka which belongs to
the Atharvaveda. c) Diamond Sutra

Select the correct answer using the codes d) Wheel Sutra


below.

a) 1 and 4 only Q 14. The first member of the


Buddhist Monastic Sangha to gain
b) 1 and 2 only Arhatship was
c) 3 only a) Ananda
d) 1, 3 and 4 only b) Sariputta
c) Kondanna
d) Mahakassapa

SHAMAN TRADITIONS

JAINISM Q 15. The doctrine of Anatta in


Q 12. Which of these was/were Buddhism means that
some of the most important ideas in 1. There is no permanent and
Jainism? independent essence in a human being
1. The entire world is fundamentally 2. The sense of self as perceived by
inanimate despite the contrary illusion. beings is illusory
2. There are no cycles of birth and death as Which of the above is/are correct?
objects come from nothingness and dissolve
into nothingness. a) 1 only
Which of the above is/are correct? b) 2 only
a) 1 only c) Both 1 and 2
b) 2 only d) None
c) Both 1 and 2

WWW.INSIGHTSIAS.COM WWW.INSIGHTSONINDIA.COM
INSIGHTSIAS PRELIMS TEST SERIES 2019 – HISTORY QUESTIONS

Q 16. Who among the following is c) 2, 4 and 5 only


a pivotal figure in the establishment
of Tibetan Buddhism and often d) 3, 4 and 5 only
called as the second Buddha?
a) Padmasambhava
Q 19. Buddhism is the dominant
b) Lawapa religion in terms of majority of
population practicing it in which of
c) Shantarakshita the following South Asian countries
d) Nagarjuna 1. Nepal
2. Bhutan
Q 17. Who was the first woman to 3. Sri Lanka
seek ordination for women in the
Buddhist tradition? Select the correct answer using the codes
below.
a) Yashodhara
a) 1 and 2 only
b) Mahapajapati Gotami
b) 2 and 3 only
c) Sujata c) 1, 2 and 3
d) Dhammananda d) 1 and 3 only

Q 18. The Buddhist literature


Anguttara Nikaya gives a list of Q 20. Mahayana refers to one of
sixteen great kingdoms called three routes to enlightenment. The
‘Sixteen Mahajanapadas’. Which of
other two routes are
the following were among them?
a) Hinayana and Vajrayana
1. Panchala
b) Upayana and Vajrayana
2. Sura
c) Hinayana and Upayana
3. Machcha
d) Azhaliism and Upayana
4. Vamsa
5. Rohini
OTHER SHAMAN TRADITIONS
Select the correct answer using the codes Q 21. With reference to the
below. Sramana movements in Ancient
Northern India, consider the
a) 1, 2 and 3 only
following statements.
b) 1, 3 and 4 only

WWW.INSIGHTSIAS.COM WWW.INSIGHTSONINDIA.COM
INSIGHTSIAS PRELIMS TEST SERIES 2019 – HISTORY QUESTIONS

1. Purana Kassapa taught that there is no TERRITORIAL STATES


virtue or sin, no merit or demerit, whatever
one does. MAHAJANAPADAS
Q 23. Which of the following
2. Ajita Kesakambali taught a form of
contributed to the strength of the
materialism, that there is no future life for us
Magadha Mahajanpada?
let alone repeated rebirth.
1. Rajagriha, which was a fortified
3. Pakudha Kaccayana held the view that
settlement, remained as the permanent
earth, water, fire, air, joy, sorrow, and life
capital of Magadha.
are stable and unproductive, independent
primordial substances. 2. Iron mines were easily accessible
providing the raw material to tool industries.
Select the correct answer using the codes
below. 3. Elephants, found in nearby forests, added
to the potential of the Magadhan army.
a) 2 only
4. Ganga and its tributaries provided a
b) 1 and 2 only means of cheap and convenient
c) 1, 2 and 3 transportation.

d) 1 and 3 only Select the correct answer using the codes


below.
a) 2 and 3 only
Q 22. What was\were the
fundamental difference(s) in the b) 1, 3 and 4 only
philosophies of Lokayatas and c) 1, 2 and 4 only
Ajivikas?
d) 2, 3 and 4
1. Ajivikas believed in fatalism, whereas
Lokayatas did not.
2. Ajivikas noted that there are no Universal Q 24. With reference to the
laws, whereas Lokayatas believed certain Mahajanpadas around the sixth
fundamental laws existed. century BCE, consider the following
statements.
Which of the above is/are correct?
1. Early Buddhist and Jaina texts
a) 1 only mention sixteen states known as
b) 2 only mahajanapadas.

c) Both 1 and 2 2. All the mahajanapadas were ruled


by kings.
d) None
3. Avanti and Kashi were two of
these Mahajanpadas.

WWW.INSIGHTSIAS.COM WWW.INSIGHTSONINDIA.COM
INSIGHTSIAS PRELIMS TEST SERIES 2019 – HISTORY QUESTIONS

Select the correct answer using the codes d) They contain Ashokan pillar inscriptions.
below.
a) 1 and 2 only
Q 27. Dhamma mahamattas, in the
b) 1 and 3 only history of India, were officials who
were appointed to administer
c) 3 only
a) Civil and Criminal justice
d) 2 and 3 only
b) Land deeds attached to religious
institutions
Q 25. The sixth century BCE is c) Translation of vedic texts
regarded as a major turning point in
early Indian history. It is associated d) None of the above
with which of the following?
1. Use of silver for the first time
POST MAURYANS
2. Development of coinage Q 28. Consider the following
3. Practice of “gana” oligarchies in statements.
mahajanpadas 1. The Ardhanarishvara, a composite
Select the correct answer using the codes androgynous form, is dated to the Kushan
below. period.

a) 2 only 2. Kalyanasundara and Panigrahana-murti


are iconographical descriptions of the
b) 1 and 3 only creation of Universe.

c) 1 and 2 only Which of the above is/are correct?

d) 2 and 3 only a) 1 only


b) 2 only

AGE OF MAURYAS c) Both 1 and 2


Q 26. What is common between d) None
the places: Rummindei, Sanchi,
Sarnath and Sahasram?
a) They are located at the same latitude. SOUTHERN KINGDOMS
b) They were the capitals of the SATAVAHANAS
Mahajanpadas.
Q 29. Consider the following
c) They are some of the earliest cities, even statements.
before the Indus valley civilization was
discovered.

WWW.INSIGHTSIAS.COM WWW.INSIGHTSONINDIA.COM
INSIGHTSIAS PRELIMS TEST SERIES 2019 – HISTORY QUESTIONS

1. Satvahanas banned the practice of b) Traditional weavers


granting tax-free villages to Brahmanas and
c) Pepper cultivators
Buddhist monks.
2. Satvahana kingdom was the first to shun d) Rich merchants
the practice of building military camps and
settlements to cut down wastage of public
money. Q 32. Yavanas are mentioned in
detail in Sangam literature refer to
Which of the above is/are correct?
a) Some Greek kingdoms
a) 1 only
b) 2 only b) Elaborate religious arrangements made by
Kings
c) Both 1 and 2
c) Horse chariots that were used for
d) None Ashvamedha sacrifices
d) Foreign conquests that lead to local
cultural assimilation
SANGAM LITERATURE
Q 30. Early Sangam texts mention
different categories of people.
Consider the following matches of Q 33. In the ancient Sangam
these categories with their social literature the sport has been called as
status. ‘Yeru thazhuvuthal’. It was also seen
in news some time before due to
1. Uzhavar : Zamindars several animal rights controversies.
It is
2. Vellalar : Local banker
a) Seval Sandai
3. Adimai : Slaves
b) Jallikattu
Select the correct answer using the codes
below. c) Killithattu
a) 1 and 2 only d) Malyutham
b) 3 only
c) 1 and 3 only GUPTA PERIOD
d) 2 only Q 34. Histories of the Gupta rulers
have sometimes been reconstructed
from Prashastis, which are

Q 31. With reference to ancient a) Compositions of praise of kings in


India, masattuvan were particular

a) Shamanic healers b) Religious textual traditions of the Gupta


empire

WWW.INSIGHTSIAS.COM WWW.INSIGHTSONINDIA.COM
INSIGHTSIAS PRELIMS TEST SERIES 2019 – HISTORY QUESTIONS

c) Administrative records of the empire NEW DECCAN STATES


d) Rock inscriptions maintained by the kings PALLAVAS
Q 37. The Pallava kings, who
succeeded the Chalukya kings in
POST GUPTAS parts of South India, were great
patrons of arts. Who among them
HARSHA AND HIS TIMES was referred to the title of
Q 35. The works of Chinese pilgrim Vichitrachitta (curious-minded)?
Xuan Zang in Buddhist texts and his a) Parameswara Varma II
travel accounts played a very crucial
role in re-discovering the essence of b) Mahendravarma I
Buddhism that was nearly lost in
c) Nandi Varma III
India. In his memory respecting his
immense contribution, the Xuan d) Danti Varma I
Zang Memorial Hall was built at
a) Rumtek Monastery
CHALUKYAS
b) Nava Nalanda Mahavihara
Q 38. Consider the following
c) Bodhgaya statements.
d) Central Pagoda, Igtapuri 1. Badami was the capital of the
western Chalukyan dynasty.
2. Badami caves are popularly known as
Q 36. With reference to Ancient Shiva caves due to the preponderance of
India, which of the following dramas Shiva images in the caves.
were written by Harsha (the monarch
mentioned in Harshacharita)? Select the correct answer using the codes
below.
1. Ratnavali
a) 1 only
2. Nagananda
b) 2 only
3. Priyardarsika
c) Both 1 and 2
Select the correct answer using the codes
below. d) None of the above
a) 1 and 2 only
b) 3 only INDIAN PHILOSOPHY
c) 1 and 3 only Q 39. Consider the following
statements.
d) 1, 2 and 3

WWW.INSIGHTSIAS.COM WWW.INSIGHTSONINDIA.COM
INSIGHTSIAS PRELIMS TEST SERIES 2019 – HISTORY QUESTIONS

1. Vishishtadvaita school of Hindu a) 1, 2 and 3 only


philosophy believes that Brahman alone
exists, and is characterized by sheer unity. b) 1 and 3 only

2. Turiya in Hindu philosophy is the c) 3 and 4 only


background that underlies and transcends
d) 1, 2, 3, 4 and 5
the common states of consciousness.
3. Bhedābheda Vedānta teaches that the
individual self is completely identical to Q 41. The Vedanta system of
Brahman. Darshana advocates that liberation is
4. Advaita Vedānta claims that the possible primarily by the means of
individual self is both different and not a) Kriya yoga
different from the ultimate reality known as
Brahman. b) Karma
Select the correct answer using the codes c) Jnana
below.
d) Bhakti
a) 1 and 2 only
b) 3 and 4 only
Q 42. Consider the following
c) 2 only statements.
d) 1 and 4 only 1. The Purusha-sukta of Rig Veda Samhita
lays the order for a classless society that
lives in peace and harmony.
Q 40. The Shakti Peeth are 2. In the Panchavimsha Brahmana, Indra is
significant shrines and pilgrimage associated with the creation of Varnas.
destinations in Shaktism, the
goddess-focused Hindu tradition. 3. The Chandogya Upanishad mentions that
Which of these Asian non-pious Brahmanas can be offered in
countries/regions host these 108 purushmedha sacrifices.
Shakti Peethas?
Select the correct answer using the codes
1. Nepal below.
2. Bangladesh a) 1 only
3. Tibet b) 2 only
4. Sri Lanka c) 1 and 3 only
5. Pakistan d) 1, 2 and 3
Select the correct answer using the codes
below.

WWW.INSIGHTSIAS.COM WWW.INSIGHTSONINDIA.COM
INSIGHTSIAS PRELIMS TEST SERIES 2019 – HISTORY QUESTIONS

TRAVELLERS AND OTHER SOCIAL LIFE d) None


Q 43. Muziris (Muchiri,
Muyirikode, Makotai,
Mahodayapuram) was an important Q 46. Consider the following about
ancient seaport and urban center on Ibn Battuta’s account of India.
the
1. The highways were safe and
a) Malabar Coast immune from robbery and other crimes.
b) Coromandal coast 2. Coconut fibre found in India was so
strong that it was used for making ropes
c) Odisha coast
which pulled ships.
d) Arikamedu Coast
3. Indian textiles such as fine muslins were
in great demand in Asian markets.

Q 44. The term ‘gahapati’ was 4. Horses and human runners were often
often used in Pali texts to designate used to dispatch goods required at short
notice.
a) Large landholders
Select the correct answer using the codes
b) Priests below.

c) Landless labourers a) 1 and 3 only

d) Court debaters b) 2 and 4 only


c) 1, 2 and 3 only

Q 45. With reference to certain d) 2, 3 and 4


works of Medieval period, consider
the following statements.
1. Al-Biruni’s Kitab-ul-Hind deals with a Q 47. Consider the following
varieties of subjects, inter alia, philosophy, statements.
alchemy, social life, laws and metrology. 1. Mauryas used punch-marked coins
2. Ibn Battuta’s book of travels, called Rihla, made of silver and copper.
written in Arabic, is a source of information 2. The first coins to bear the names and
about the social and cultural life in the images of rulers were issued by the Indo-
Indian subcontinent of the Medieval period. Greeks.
Which of the above is/are correct? 3. Yaudheyas of Punjab and Haryana were
a) 1 only renowned for issuing copper coins.

b) 2 only 4. Both Kushanas and Guptas issued gold


coins.
c) Both 1 and 2

WWW.INSIGHTSIAS.COM WWW.INSIGHTSONINDIA.COM
INSIGHTSIAS PRELIMS TEST SERIES 2019 – HISTORY QUESTIONS

Select the correct answer using the codes a) Unani


below.
b) Siddha
a) 1 and 2 only
c) Sowa Rigpa
b) 2, 3 and 4 only
d) Ayurveda
c) 1, 3 and 4 only
d) 1, 2, 3 and 4
Q 50. Consider the following
statements about Indian systems of
Medicine (ISM).
Q 48. Consider the following
statements with reference to 1. The preventive aspect of Ayurveda is
developments in Medieval India. called Svasth-Vritta and includes personal
hygiene, daily and seasonal regimens.
1. A personal astronomical observatory was
built by Humayun on the banks of Yamuna 2. The curative treatment aspect of
River. Ayurveda consists of Aushadhi, Ahara and
Vihara.
2. Metal cylinder rockets were often used in
wars waged by some of the Mughal rulers. Which of the above is/are correct?
3. Techniques related to making of a) 1 only
shampoo and related alkali based soaps were
introduced in Mughal India from Europe. b) 2 only

Select the correct answer using the codes c) Both 1 and 2


below.
d) None
a) 1 and 2 only
b) 2 and 3 only
Q 51. Consider the following
c) 1 only statements about Indian systems of
Medicine (ISM).
d) 1, 2 and 3
1. The Agnivesha Tantra was edited
by Charaka.
Q 49. This system of medicine 2. ‘Brahma Samhita’ was an
became popular after 7th CE with the important treatise of Ayurveda.
approach of Buddhism to Tibet. The
fundamental text book of this 3. Bhela Samhita, the treatise containing the
medicine was first translated from most Ayurvedic formulations, has not
India and enriched in Tibet with its survived.
own folklore and other medical
Select the correct answer using the codes
tradition like Chinese and Persian
below.
etc. It is

WWW.INSIGHTSIAS.COM WWW.INSIGHTSONINDIA.COM
INSIGHTSIAS PRELIMS TEST SERIES 2019 – HISTORY QUESTIONS

a) 2 only c) 2 and 3 only


b) 1 and 2 only d) 3 only
c) 2 and 3 only
d) 1, 2 and 3 only SOCIAL LIFE AND DEVELOPMENT
Q 54. Sarais were an interesting
feature of medieval India. These
were meant to provide
MEDIEVAL INDIAN HISTORY a) Temporary accommodation
BHAKTI MOVEMENT b) Debate forums
Q 52. Sufi Silsilas became a major
tradition in Medieval India. The term c) Courtyard for general purposes
silsila implies d) Sub-urban expansion space
a) A family of guest houses for spiritual
debates
VIJAYANAGARA EMPIRE
b) A spiritual genealogy linking the master
to the disciples Q 55. The royal title of
“Devaputra” and “Hindu Suratrana”
c) A method of chanting leading to are often associated with which of
communion with God these ruler/kingdoms respectively?
d) A distinct way of Sufi poetry that later a) Kushanas and Vijayanagara
developed as a mature Classical style
b) Magadha and Samudragupta
c) Shakas and Cholas
Q 53. Consider the following about
d) Vakatakas and Cheras
Alvars and Nayanars.
1. They disapproved of the caste
based discrimination in India. Q 56. Consider the following
statements.
2. They did not produce any written
compositions. 1. Portuguese travellers Domingo Paes and
Fernao Nuniz visited his Empire.
3. Women devotees were not
allowed within the Sangha. 2. He defeated the Sultans of Bijapur,
Golconda, the Bahmani Sultanate and the
Select the correct answer using the codes
Raja of Odisha.
below.
3. The south Indian mathematician
a) 1 and 2 only
Nilakantha Somayaji lived in his Empire.
b) 1 only
The above refer to?

WWW.INSIGHTSIAS.COM WWW.INSIGHTSONINDIA.COM
INSIGHTSIAS PRELIMS TEST SERIES 2019 – HISTORY QUESTIONS

a) Sadasiya Raya CONSOLIDATION OF MUGHALS


b) Krishnadevaraya Q 59. Shiqdar officials in Mughal
India were mainly responsible for
c) Venkata I
a) Regulating water storage and availability
d) Thimma Bhupala in the provinces
b) Maintaining law and order

Q 57. Consider the following about c) Implementing fatwas issued by the


the Vijayanagara Empire. emperor

1. Chinese porcelains were banned within d) Supervising work of people welfare


the Empire by the rulers to promote local departments
craftsmanship.
2. Arrangements were made to store
rainwater and conduct it to the city. Q 60. Consider the following with
reference to the forest Society and
3. Fortifications around the Empire enclosed tribes in the Mughal Period.
agricultural tracts.
1. Forest tribes often supplied
Select the correct answer using the codes elephants to the state as a tribute.
below.
2. Forests provided gum lac which was a
a) 1 and 2 only major export item overseas from India.
b) 2 and 3 only 3. The state banned private commercial
agriculture in forest regions to preserve their
c) 1 only pristine ecology.
d) 1, 2 and 3 Select the correct answer using the codes
below.
a) 1 and 2 only
STRUGLE FOR EMPIRE IN NORTH INDIA
Q 58. The Ahom kings followed a b) 2 and 3 only
peculiar practice where people were
c) 3 only
obliged to render military service in
exchange for land. The practice was d) 1, 2 and 3
popular in the present day state of
a) Meghalaya
Q 61. Consider the following
b) Nagaland statements.
c) Sikkim 1. Expansion of foreign trade in the Mughal
d) Assam Empire often involved the use of silver
bullions as payment.

WWW.INSIGHTSIAS.COM WWW.INSIGHTSONINDIA.COM
INSIGHTSIAS PRELIMS TEST SERIES 2019 – HISTORY QUESTIONS

2. Silver coins were discontinued in British In the context of the above, which of these is
India in view of growing deficit of the correct?
precious metal.
a) A is correct, and R is an appropriate
Which of the above is/are correct? explanation of A.
a) 1 only b) A is correct, but R is not an appropriate
explanation of A.
b) 2 only
c) A is correct, but R is incorrect.
c) Both 1 and 2
d) Both A and R are incorrect.
d) None

Q 62. Consider the following AKBAR


matches of important officials in the Q 64. Consider the following about
Mughal Empire and their sulh-i-kul introduced during Akbar’s
responsibilities. regime.

1. Fotedar : Treasurer 1. The ideal of sulh-i kul was implemented


through state policies.
2. Diwan-i-Rasalat : Army Commander
2. Sulh-i-kul allowed all religions and
3. Karkuns : Engineering structures schools of thought to flourish provided they
don’t impinge on state authority.
4. Diwan-i-Insha : Minister for
Communications Which of the above is/are correct?
Select the correct answer using the codes a) 1 only
below.
b) 2 only
a) 1 and 4 only
c) Both 1 and 2
b) 2 and 3 only
d) None
c) 1, 3 and 4 only
d) 1, 2 and 3 only
LIFE AND DEVELOPMENT
Q 65. Consider the following
Q 63. Consider the following about
the Mansabdari System under the terminologies used in Medieval India
Mughal administration. and its depiction.

Assertion (A): The Mansabdari system was 1. Parwana was an order issued by
organized on hereditary lines. higher authorities.
2. Siyaha huzur was proceedings of
Reason (R): The property of a Mansabdar
was not confiscated after his death. the provincial court.

WWW.INSIGHTSIAS.COM WWW.INSIGHTSONINDIA.COM
INSIGHTSIAS PRELIMS TEST SERIES 2019 – HISTORY QUESTIONS

3. Andarz-o-chihra were documents 3. The panchayat could derive its funds from
on personnel and horses. contributions made by community residents
to a common financial pool.
4. Roznamcha-i-waqai was daily news
report. Select the correct answer using the codes
below.
Select the correct answer using the codes
below. a) 1 and 2 only
a) 1 and 3 only b) 3 only
b) 2 and 4 only c) 1 and 3 only
c) 1, 2 and 3 only d) 2 and 3 only
d) 1, 2, 3 and 4

Q 68. Consider the following terms


Q 66. Some sources of the in the context of Medieval India and
seventeenth century refer to two their correct matches.
terms – khud-kashta and pahi-kashta. 1. Jajmani system: Art of craft
They refer to production
1. Types of absentee landlordism in rural 2. Mallahzadas: Cattle traders
parganas
3. Milkiyat: Private Land of Zamindars
2. Land grants given by the Amin to notable
individuals Select the correct matches using the codes
below.
Which of the above is/are correct?
a) 1 only
a) 1 only
b) 2 and 3 only
b) 2 only
c) 3 only
c) Both 1 and 2
d) 1 only
d) None

MODERN INDIAN HISTORY


Q 67. Consider the following about
rural institutions in Medieval India. BRITISH EXPANSION AND
1. The village panchayats represented only CONSOLIDATION
the most dominant caste in the village. Q 69. The Battle held at Plassey is
often mentioned in India’s colonial
2. The village headman was to be history. How did the place Plassey
compulsorily nominated by the Subedar who get its name from?
headed the Parganas.
a) Palash tree that grow there

WWW.INSIGHTSIAS.COM WWW.INSIGHTSONINDIA.COM
INSIGHTSIAS PRELIMS TEST SERIES 2019 – HISTORY QUESTIONS

b) The water structures that surrounded a) 1 only


large forts
b) 2 only
c) Highly fertile soil found there
c) Both 1 and 2
d) Series of battles fought at it
d) None

OTHERS
Q 72. The Durand Commission
Q 70. Consider the following (1893) was set up to
statements about certain events
pertaining to the 17th CE, colonial a) Recommend changes in the proposed
India. Universities Act
1. The East India Company had purchased b) Bring reforms in Police administration in
the right of settlement in Madraspatam from India on the lines suggested by Lord Curzon
the local Telugu lords, the Nayaks of
Kalahasti despite their vehement opposition. c) Define the Durand Line between India
and Afghanistan (now between Pakistan and
2. Rivalry (1746-63) with the French East Afghanistan)
India Company led the British to flee
Madras and appoint residents in the princely d) Increase the intake of British army in
courts of Madras. India from contingents stationed abroad

Which of the above is/are correct?


a) 1 only THE REVOLT OF 1857
Q 73. Who served as the last kotwal
b) 2 only
of Delhi (Chief of police) in the
c) Both 1 and 2 court of the Mughal emperor
Bahadur Shah II, before the position
d) None was abolished following the Indian
Rebellion of 1857?
a) Ghaznafar Khan
Q 71. Consider the following
statements. b) Mir-i-Atish

Younghusband's Mission to Tibet (1904) c) Ganga Dhar Nehru

1. Led to a temporary invasion by British d) Rai Pithora


Indian forces under the auspices of the Tibet
Frontier Commission
2. Intended to establish diplomatic relations Q 74. In the context of Modern
and resolve the dispute over the border History of India, the Azamgarh
between Tibet and Bhutan Proclamation is generally associated
with
Which of the above is/are correct?

WWW.INSIGHTSIAS.COM WWW.INSIGHTSONINDIA.COM
INSIGHTSIAS PRELIMS TEST SERIES 2019 – HISTORY QUESTIONS

a) Sepoy Mutiny 1857 b) 2, 3 and 4 only


b) Swadeshi Movement c) 1, 3 and 4 only
c) Rowlatt Satyagraha d) 1, 2, 3 and 4
d) Champaran Movement

Q 77. Which of these places were


known for hosting revolts in the
British Indian Army before the 1857
revolt happened?
Q 75. Awadh would not have been
annexed by the British in 1856 if the 1. Vellore
Nawab of Awadh had
2. Barrackpore
1. A natural heir to trespass the
Doctrine of Lapse 3. Singhbhum

2. Not refused to introduce reforms Select the correct answer using the codes
as suggested by the British below.

Which of the above is/are correct? a) 1 and 2 only

a) 1 only b) 2 and 3 only

b) 2 only c) 1 and 3 only

c) Both 1 and 2 d) 1, 2 and 3

d) None
Q 78. The assumption of the
Government of India by the
Q 76. The revolt of 1857 nearly sovereign of Great Britain, after the
coincided or followed certain events 1857 revolt, was announced
in which the British suffered serious
losses or were involved for major 1. By Lord Dufferin
gains; these included?
2. At a durbar at Allahabad
1. The First Afghan War
3. In the 'Queen's Proclamation'
2. Punjab Wars (1845-49) issued in 1857

3. Crimean Wars Select the correct answer using the codes


below.
4. Santhal rebellion
a) 1 only
Select the correct answer using the codes
below. b) 2 only

a) 1 and 2 only c) 2 and 3 only

WWW.INSIGHTSIAS.COM WWW.INSIGHTSONINDIA.COM
INSIGHTSIAS PRELIMS TEST SERIES 2019 – HISTORY QUESTIONS

d) 3 only movement and led the anti-Brahmin


movement.
2. Madurai Pillai, affectionately called
Q 79. In the context of Modern Thatha, for his untiring efforts towards
History of India, the Azamgarh scheduled castes, accompanied Dr.
Proclamation is generally associated Ambedkar to the Round Table Conference.
with
Which of the above is/are correct?
a) Sepoy Mutiny 1857
a) 1 only
b) Swadeshi Movement
b) 2 only
c) Rowlatt Satyagraha
c) Both 1 and 2
d) Champaran Movement
d) None

Q 80. Queen Victoria’s


Proclamation of 1858 Q 82. Which of these personalities
were associated with Brahmo Samaj?
1. Assured the Indian people equal and
impartial protection of law and freedom of 1. Debendranath Tagore
religion and social practices.
2. Keshub Chandra Sen
2. Announced the end of East India
Company’s rule in India and the Queen’s 3. Pronob Bhattacharjee
assumption of the Government of India Select the correct answer using the codes
Which of the above is/are correct? below.

a) 1 only a) 1 and 2 only

b) 2 only b) 2 only

c) Both 1 and 2 c) 2 and 3 only

d) None d) 1 and 3 only

Q 83. Consider the following about


the history of the Justice party –
RELIGIOUS AND SOCIAL REFORM Dravidian Movement – and its
ideology.
MOVEMENTS
Q 81. Consider the following 1. Periyar E. V. Ramaswamy
statement. transformed the Justice Party into the social
organisation Dravidar Kazhagam.
1. E.V. Ramasami Naicker was a worker of
the Congress party, started the self-respect

WWW.INSIGHTSIAS.COM WWW.INSIGHTSONINDIA.COM
INSIGHTSIAS PRELIMS TEST SERIES 2019 – HISTORY QUESTIONS

2. The party helped established the Q 85. Consider the following


non-cooperation movement in South India statements.
with the support of C. Rajagopalachari.
1. In Bombay, the Paramhans Mandali was
3. The organization opposed Annie founded in 1940 to work for the abolition of
Besant and her Home rule movement. caste.
Select the correct answer using the codes 2. The Satnami movement in Central India
below. was founded by Ghasidas who worked
among the leatherworkers and organised a
a) 2 only movement to improve their social status.
b) 1, 2 and 3 only Which of the above is/are correct?
c) 1 and 3 only a) 1 only
d) 1 and 2 only b) 2 only
c) Both 1 and 2
d) None
Q 84. Consider the following
statements
Q 86. Consider the following
1. The Sarda Act (1930) pushed up the statements.
marriage age to 18 and 14 for boys and girls
respectively. 1. Deoband movement aimed at the welfare
of Muslims through western education and
2. The Bengal regulations of 1795 and 1804 support of the British Government.
declared infanticide illegal and equivalent to
murder. 2. Aligarh movement emphasised
reconciliation of Islamic teachings with the
3. The Age of Consent Act (1891) needs of the modern age.
which forbade the marriage of girls below
the age of 12. Which of the above is/are correct?

Assuming the dates for these a) 1 only


acts/regulations to be correct, which of the
above is/are correct? b) 2 only

a) 1 and 2 only c) Both 1 and 2

b) 3 only d) None

c) 1 and 3 only
d) 1, 2 and 3 Q 87. Consider the following
statements.

WWW.INSIGHTSIAS.COM WWW.INSIGHTSONINDIA.COM
INSIGHTSIAS PRELIMS TEST SERIES 2019 – HISTORY QUESTIONS

1. The Brahmo Samaj, formed in Q 89. Consider the following


19 CE, prohibited all forms of idolatry and
th
statements about popular socio-
sacrifice. religious movements in Northern
part of India.
2. Henry Louis Vivian Derozio, a teacher at
Hindu College, Calcutta, in the early 19th 1. The Singh Sabhas were aimed at
CE, encouraged his pupils to question all reforming the Sikh society.
authority.
2. The Nirankari Movement insisted on the
3. The Prarthana Samaj sought to remove worship of formless God.
caste restrictions, abolish child marriage,
encourage the education of women and 3. The Namdhari Movement followers wore
rejected the Hindu and Christian texts. white clothes and practiced vegetarianism.

Select the correct answer using the codes Select the correct answer using the codes
below. below.

a) 1 and 2 only a) 2 only

b) 2 and 3 only b) 1 and 3 only

c) 1 and 3 only c) 2 and 3 only

d) 1, 2 and 3 d) 1, 2 and 3 only

Q 88. Consider the following Q 90. People in the Ezhava


statements. community in Kerala, at the start of
the 20th CE, were led by him to
1. He relied upon legislation to do change their social practices:
away with social ills.
a) Narayana Guru
2. To encourage consideration of social
problems on a national scale, he inaugurated b) E. V. Ramaswamy Naicker
the Indian National Social Conference
c) Swami Vivekananda
(INSC).
d) T. K. Madhavan
The above refers to?
a) Keshub Chandra Sen
b) Baba Amte Q 91. Consider the following
statements.
c) Mahadev Govind Ranade
1. The Swadhyay Parivar is a devotional
d) Arbondo Ghosh movement started by Pandurang Shastri
Athavale to promote study of the ‘self’.
2. The Swadhyay Parivar movement lays
exclusive emphasis on one’s own faculty for

WWW.INSIGHTSIAS.COM WWW.INSIGHTSONINDIA.COM
INSIGHTSIAS PRELIMS TEST SERIES 2019 – HISTORY QUESTIONS

understanding the true nature of things and Which of the above is/are correct?
does not promote scriptural knowledge or
studies. a) 1 only

Which of the above is/are correct? b) 2 only

a) 1 only c) Both 1 and 2

b) 2 only d) None

c) Both 1 and 2
d) None INDIAN NATIONAL CONGRESS AIMS AND
OBJECTIVES
Q 94. Consider the following about
the Indian National Congress (INC),
THE STRUGGLE BEGINS a party that spearheaded and helped
organize the freedom movement.
PRE-CONGRESS
Q 92. Consider the following early 1. The first president of INC was
political associations in British India. Womesh Chandra Banerji.
1. The British Indian Association 2. Mahatma Gandhi presided over any INC
2. The Bombay Association session only after independence.

3. East India Association, London 3. The first Englishman to become the


president of INC was A. O. Hume.
4. Poona Sarvojanik Sabha
4. The president of INC at the time of India's
What is the correct chronological order in which
independence was Jawahar Lal Nehru.
they were formed?
a) 1234 Select the correct answer using the codes
below.
b) 4321
a) 1 and 4 only
c) 1432
b) 2 and 3 only
d) 2314
c) 1 only
d) 2, 3 and 4 only
Q 93. The Ilbert Bill Controversy is
a high watermark in the history of
Indian National Movement. It is
related to Q 95. Arrange these events in the
Modern Indian history
1. Improving working conditions in chronologically.
factories for Indian workers
1. Establishment of the Indian
2. Ban of indigenous press and National Congress
missionary activities by the state

WWW.INSIGHTSIAS.COM WWW.INSIGHTSONINDIA.COM
INSIGHTSIAS PRELIMS TEST SERIES 2019 – HISTORY QUESTIONS

2. Appointment of Police Commission under 1. Aurobindo Ghosh


Sir Andrew Frazer to review police
administration 2. Raj Narain Bose

3. The Ilbert Bill controversy 3. Vishnu Shastri Chiplunkar


4. Surendranath Banerjee
4. The Second Afghan War
Select the correct answer using the codes
Select the correct answer using the codes
below.
below.
a) 1 and 2 only
a) 3124
b) 2, 3 and 4 only
b) 4321
c) 1 and 4 only
c) 4312
d) 1, 2 and 3 only
d) 1342

Q 98. The 'Alipore Bomb Case' is a


NATIONAL MOVEMENT 1905-1918 notable event in the Modern Indian
History because
MITITANT NATIONALISM
1. It was the first state trial of such a
Q 96. The Ghadr part was
high magnitude in India
motivated to wrestle freedom from
the British due to which of these 2. It was the first attempt of any
events? Pick out the possible events extremist at overthrowing the state.
that could have influenced their
activism. Which of the above is/are correct?
1. Komagata Maru Incident a) 1 only
2. Outbreak of First World War b) 2 only
Which of the above is/are correct? c) Both 1 and 2
a) 1 only d) None
b) 2 only
c) Both 1 and 2 SWADESHI AND BOYCOTT
Q 99. With reference to the
d) None
Congress split 1907, consider the
following statements.

Q 97. Who among the following 1. The moderates supported the resolutions
can be associated with a militant on Swaraj, Swadeshi and Boycott of foreign
goods as proposed by extremists but differed
approach to political work towards
in the approach to implement these ideals.
Indian Independence?

WWW.INSIGHTSIAS.COM WWW.INSIGHTSONINDIA.COM
INSIGHTSIAS PRELIMS TEST SERIES 2019 – HISTORY QUESTIONS

2. In the Surat Session, extremists wanted Select the correct answer using the codes
Lala Lajpat Rai or Bal Gangadhar Tilak as a below.
President candidate of congress whereas
Moderates supported Dr. Rashbihari Ghosh. a) 1 and 2 only

Which of the above is/are correct? b) 1 and 3 only


c) 2 and 3 only
a) 1 only
d) 3 only
b) 2 only
c) Both 1 and 2
EXTREMIST LEADERSHIP
d) None
Q 102. Consider the following about
Rowlatt Act.

Q 100. Dadabhai Naoroji's declared 1. It allowed the British government


at the “X” session of Indian National to imprison people without due trial.
Congress that self-government or
swaraj was to be the goal of the 2. The Act banned all Indian owned
Congress. X is presses from circulating news.

a) Calcutta session (1906) 3. It was repealed following the


Jallianwala Bagh massacre.
b) Lucknow Session (1916)
Select the correct answer using the codes
c) Lahore Session (1929) below.
d) Benaras (1912) a) 1 only

b) 2 and 3 only
Q 101. Why did the Swadeshi c) 1 and 2 only
movement fizzle out? Pick out
possible reasons from the choices d) 1 and 3 only
given below:
1. There was severe government
repression. Q 103. Consider the following
events occurring around the setting
2. It rejected the techniques that later came up of Home Rule Movement in
to be associated with successful Gandhian India.
politics such as noncooperation and passive
resistance. 1. By early 1915, Annie Besant had
launched a campaign to demand self
3. Pesantry did not take part in large government for India after the war on the
numbers and the movement was largely lines of white colonies.
restricted to the upper social and economic
classes.

WWW.INSIGHTSIAS.COM WWW.INSIGHTSONINDIA.COM
INSIGHTSIAS PRELIMS TEST SERIES 2019 – HISTORY QUESTIONS

2. Tilak and Annie Besant set up two Q 105. The Lucknow Pact, 1916,
different Home Rule leagues after separating between Congress and Muslim
from a common Home Rule movement. League brought them together.
happened at a time when the Muslim
3. In 1915 it was decided that the League was gradually turning anti-
Extremists be admitted to the Congress. imperialist. What could have been
the possible factors behind such an
4. Leaders such as Motilal Nehru and
attitude of the Muslim League?
Jawaharlal Nehru completely boycotted the
movement. 1. Britain's refusal to help Turkey in its wars
in the Balkans (1912-13) and with Italy
Select the correct answer using the codes
(during 1911) had infuriated the Muslims
below.
2. Annulment of the partition of Bengal in
a) 1 and 3 only
1911 was not well received by all sections of
b) 2 and 4 only the Muslims leaders

c) 2, 3 and 4 only Which of the above is/are correct?

d) 1, 3 and 4 only a) 1 only


b) 2 only

Q 104. Which of the following c) Both 1 and 2


beliefs defined the extremists in the d) None
Indian National Movement?
1. Belief in England's providential
mission in India Q 106. Which of these personalities
was/were associated with founding
2. Demanded constitutional reforms
of the Home Rule Movement
and share for Indians in services
(1916)?
3. Believed that political connections with
1. Annie Besant
Britain would perpetuate British
exploitation of India 2. Madame Blavtasky
Select the correct answer using the codes 3. A.O. Hume
below.
4. Bal Gangadhar Tilak
a) 3 only
Select the correct answer using the codes
b) 2 and 3 only below.
c) 1 and 2 only a) 1 and 2 only
d) 1 and 3 only
b) 2 and 4 only
c) 1 and 4 only

WWW.INSIGHTSIAS.COM WWW.INSIGHTSONINDIA.COM
INSIGHTSIAS PRELIMS TEST SERIES 2019 – HISTORY QUESTIONS

d) 1, 2 and 3 only of self-governing institutions with a view to


the progressive realisation of responsible
government in India as an integral part of
the British Empire". This statement coming
Q 107. The Lahore Session of the from the then Secretary of State is closely
Indian National Congress (INC) was associated with which of these events
significant for which of the
following reasons? a) Sepoy Mutiny, 1857
1. Election of M.K. Gandhi as INC President b) First World War (1914-18)
2. Proclamation of commitment to Purna c) Second World War (1939-45)
Swaraj
d) Incoming of the Simon Commission,
Which of the above is/are correct? 1929
a) 1 only
b) 2 only Q 110. With reference to the First
c) Both 1 and 2 World war and nationalist response,
consider the following statements.
d) None
1. While the moderates supported India’s
participation in the war, the extremists
rejected any such involvement.
Q 108. During the colonial era, the
“India League” was a Britain-based 2. The British government promised self-
organization whose main aim was to government as a tribute to India’s
participation in the war.
a) Eradicate untouchability from India
Which of the above is/are correct?
b) Increase the representation of Indian MPs
in British Parliament a) 1 only

c) Work for the welfare of Indian b) 2 only


immigrants in Britain c) Both 1 and 2
d) Achieve full independence and self- d) None
government for India

WORLD WAR 1 and Indian response


Q 109. Consider this statement. BRITISH RESPONSE
Q 111. Consider the following
“The Policy of His Majesty's Government, statements.
with which the Government of India are in
complete accord, is that of increasing 1. In 1911, Delhi became capital of India,
association of Indians in every branch of the headed by a Commissioner and then known
administration and the gradual development as “Chief Commissioner’s Province.”

WWW.INSIGHTSIAS.COM WWW.INSIGHTSONINDIA.COM
INSIGHTSIAS PRELIMS TEST SERIES 2019 – HISTORY QUESTIONS

2. As per the Constitution, barring a few d) 1 and 4 only


matters, the Delhi assembly can legislate on
all matters listed in the State List and
Concurrent List, as applicable to union
Q 113. What importance does the
territories.
place Pietermaritzburg, South Africa,
Which of the above is/are correct? hold in Gandhiji’s life?

a) 1 only a) It was from here that he launched his first


Satyagraha.
b) 2 only
b) This was the first place where he was
c) Both 1 and 2 imprisoned for protesting against the
government.
d) None
c) He was thrown off the train’s first class
compartment at this station which led him to
work against racial discrimination.
NATIONAL MOVEMENT 1919-1939
d) His first major nationalist writing was
EMERGENCE OF GANDHI based on events held in this place, that later
Q 112. ‘Swaraj’ is an analogous find a mention in his book, “Truth is God”
concept to Freedom in Indian
political thought is. Which of the
following are the ideas related to
Swaraj, as outlined by Gandhiji? Q 114. Consider the following about
Gandhian ideas of non-violence or
1. It is swaraj when we learn to rule Ahimsa.
ourselves.
1. Non-violence meant not just refraining
2. Swaraj is not just freedom but liberation from causing physical or mental harm to
in redeeming one’s self-respect and someone but also the thought of harming
capacities for self-realisation from someone.
institutions of dehumanisation.
2. The practice of non-violence is not a
3. Understanding the real ‘Self ’, and its practice of passive spiritualism but that of
relation to communities and society, is conscious compassion.
critical to the project of attaining Swaraj.
Which of the above is/are correct?
4. Sacrificing oneself for a larger cause is
Swaraj. a) 1 only

Select the correct answer using the codes b) 2 only


below. c) Both 1 and 2
a) 1, 2 and 3 only d) None
b) 1, 2, 3 and 4
c) 2 and 3 only

WWW.INSIGHTSIAS.COM WWW.INSIGHTSONINDIA.COM
INSIGHTSIAS PRELIMS TEST SERIES 2019 – HISTORY QUESTIONS

Q 115. Consider the following about 1. 12th March, 1930: Gandhi began his famous
use of the term “Sarvodaya” in India. March to Dandi from Sabarmati

1. It was associated with the translation of 2. 5th April, 1930: Gandhi reached the coast of
John Ruskin's tract on political economy, Dandi
Unto This Last, by Gandhi. 3. 6th April, 1930: Salt Law broken at Dandi
2. It was the name for a social Based on the above timeline, when was the Civil
movement in post-independence India. Disobedience movement formally launched?

3. It was used as a name to call the a) At Sabarmati on 12th March


tirtha of Mahavira.
b) At Dandi on 5th April
Select the correct answer using the codes c) At Dandi on 6th April
below.
d) At Mumbai on a later date after approval
a) 1 and 2 only from the Indian National Congress

b) 2 and 3 only
c) 3 only Q 118. Consider the following
statements about the role of
d) 1, 2 and 3 Mahatma Gandhi and his comrades
in promoting the national movement.
1. A series of “Praja Mandals” were
established to promote the nationalist creed
Q 116. Consider the following in the princely states.
statements. 2. Gandhiji encouraged the communication
1. Gandhiji spent a year travelling around of the nationalist message in English
British India on Gokhale’s advice. language, rather than Hindi or mother
tongue, to make the message accessible to
2. Gandhiji first forged the techniques of all.
non-violent satyagraha in South Africa.
3. Provincial committees of the Congress
Which of the above is/are correct? were based on the administrative divisions
(states) carved by British India; a linguistic
a) 1 only basis was avoided to curb regional
tendencies.
b) 2 only
Select the correct answer using the codes
c) Both 1 and 2
below.
d) None
a) 1 only
b) 2 and 3 only
Q 117. Consider the following timeline. c) 1 and 3 only
d) 1, 2 and 3

WWW.INSIGHTSIAS.COM WWW.INSIGHTSONINDIA.COM
INSIGHTSIAS PRELIMS TEST SERIES 2019 – HISTORY QUESTIONS

3. be a habitual khadi wearer and


spinner
Q 119. At Gandhiji’s speech at
Banaras in February 1916 at the Select the correct answer using the codes
opening of the Benaras Hindu below.
Vishwavidyalaya (BHU), he
conveyed the message that a) 1 only

1. There is no salvation for India unless the b) 2 and 3 only


rich and the elite become the trustees for c) 3 only
their countrymen.
d) 1, 2 and 3
2. There can be no spirit of self-government
about Indians if they take away from the
peasants the results of their labour.
Q 122. Fasting was made a major
Which of the above is/are correct? instrument for Satygraha in British
India by Gandhi. He undertook fast
a) 1 only
unto death for which of these causes?
b) 2 only
a) Ahmedabad Mill Workers strike
c) Both 1 and 2 b) Communal Award that gave separate
d) None electorate to untouchables
c) In 1947 for communal harmony

Q 120. Gandhiji represented the d) All of the above


Indian National Congress in which
of these Round Table Conferences?
NON-COOPERATION MOVEMENT
a) First Round Table Conference, 1930-31
Q 123. Consider the following about
b) Second Round Table Conference, 1931 the Khilafat Movement.

c) Third Round Table Conference, 1932 1. It was initiated by Maulana Abdul


Kalam Azad.
d) None of the above
2. It demanded that Khalifa must retain
control over the Muslim sacred places in the
erstwhile Ottoman Empire.
Q 121. Gandhiji listed seven rules as
“essential for every Satyagrahi in 3. The Congress boycotted the
India”. These rules included that the movement due to its violent moorings.
Satyagrahi must
Select the correct answer using the codes
1. have living faith in God below.
2. have faith in the inherent goodness a) 1 and 2 only
of human nature

WWW.INSIGHTSIAS.COM WWW.INSIGHTSONINDIA.COM
INSIGHTSIAS PRELIMS TEST SERIES 2019 – HISTORY QUESTIONS

b) 2 and 3 only d) Both A and R are incorrect.


c) 2 only
d) 1 and 3 only NEHRU REPORT
Q 126. The Nehru Report of 1928
1. Was a memorandum outlining a
Q 124. Consider the following
proposed new dominion status constitution
statements.
for India
1. M.K. Gandhi issued a manifesto in March
2. Was prepared by a committee of the
1920, announcing his doctrine of non-
Indian National Congress (INC)
violent Non-Cooperation Movement.
3. Contained a Bill of Rights unlike the
2. C.R. Das moved the resolution on non-
Government of India Act, 1935 that was
cooperation in the annual session of the
passed later
Congress in Nagpur in 1920.
Select the correct answer using the codes
Which of the above is/are correct?
below.
a) 1 only a) 1 and 2 only
b) 2 only b) 2 and 3 only
c) Both 1 and 2 c) 1 and 3 only
d) None d) 1 only

ANTI SIMON MOVEMENT Q 127. Which of the following


Q 125. Consider the following situations led to the Congress
statements. constituting a committee for that
ultimately produced the Nehru report
Assertion (A): All political groups in 1928?
decided to boycott Simon Commission.
a) Lord Montague and Chelmsford gave an
Reason (R): Simon Commission has assurance to the Indian National Congress
no Indian member. (INC) that their draft constitution would be
admitted by the British government
In the context of the above, which of these
is correct? b) The Morley Minto Act 1919 provided for
the establishment of an Indian committee
a) A is correct, and R is an appropriate that would determine the future course of
explanation of A. constitutional reforms

b) A is correct, but R is not an appropriate c) The Simon Commission exhorted Indians


explanation of A. to produce an all inclusive constitution that
will be considered by the British in the first
c) A is incorrect, but R is correct. Round Table Conference.

WWW.INSIGHTSIAS.COM WWW.INSIGHTSONINDIA.COM
INSIGHTSIAS PRELIMS TEST SERIES 2019 – HISTORY QUESTIONS

d) The then Secretary of State, Lord Q 130. The Red Shirts movement
Birkenhead, challenged the Indians to was launched by
produce a Constitution that would be
acceptable to all. a) Khan Abdul Gaffar Khan

CIVIL DISOBEDIENCE MOVEMENT b) P. Krishna Pillai


Q 128. The Dandi March was c) Sawai Tendulkar
launched against
d) Maulana Abul Kalam Azad
1. State monopoly on manufacture and sale
of salt
Q 131. Consider the following
2. Exorbitantly high salt tax
statements.
Which of the above is/are correct?
1. C. Rajagopalachari led a salt march from
a) 1 only Trichinopoly to Vedaranniyam on the
Tanjore coast in Tamil Nadu, in support of
b) 2 only the Civil Disobedience Movement.

c) Both 1 and 2 2. K. Kelappan, a Nair Congress leader,


launched the Vaikom Satyagraha and
d) None marched from Calicut to Payanneer in
defiance of salt laws.
Which of the above is/are correct?
Q 129. The Karachi Session of the
Indian National Congress is famously a) 1 only
known for
b) 2 only
1. Adopting the Nehru report c) Both 1 and 2
2. A concrete Fundamental Rights d) None
and Economic Programme

3. Endorsing the Gandhi Irwin Pact


Q 132. The Civil Disobedience
Select the correct answer using the codes Movement was called off by which of
below. the following agreements/events?
a) 1 and 2 only a) Gandhi-Irwin Pact

b) 2 and 3 only b) August Offer


c) Cabinet Mission
c) 3 only
d) Cripps Mission
d) 2 only

WWW.INSIGHTSIAS.COM WWW.INSIGHTSONINDIA.COM
INSIGHTSIAS PRELIMS TEST SERIES 2019 – HISTORY QUESTIONS

ROUND TABLE CONFERENCE Q 135. The Socialist Party went


Q 133. Gandhi opposed separate through many splits and reunions
electorates because he believed leading to the formation of many
socialist parties. These included the
1. It would mean perpetual bondage of the
harijans. 1. Nav Mazdoor Sangh

2. It would lead to social disharmony and 2. Kisan Mazdoor Praja Party


lack of unity.
3. Praja Socialist Party
Which of the above is/are correct?
Select the correct answer using the codes
a) 1 only below.

b) 2 only a) 1 only

c) Both 1 and 2 b) 2 and 3 only

d) None c) 1 and 3 only


d) 1, 2 and 3

OTHER EVENTS AND MISC


Q 134. Consider the following
Q 136. Consider the following about
statements about major
Congress socialist party.
events/occurrences in 1930-1940s.
1. It was formed within the Congress
1. Poona Pact gave away the Hindu joint
immediately after the failure of the Non-
electorate and reserves special electorates to
cooperation movement.
the depressed classes.
2. It rejected the ideology of democracy and
2. C. Rajagopalachari became the first
supported a Maoist communist revolution.
governor-general of the Dominion of India.
3. It later restructured itself as the All India
3. The members of the interim government
Kisan Sabha (AIKS).
were members of the Viceroy’s Executive
Council, and the council came to be vice- Select the correct answer using the codes
chaired by Pandit Nehru. below.
Select the correct answer using the codes a) 1 only
below.
b) 2 and 3 only
a) 3 only
c) 3 only
b) 2 and 3 only
d) None of the above
c) 1 and 2 only
d) 2 only
Q 137. In 1931, the Karachi Session
of the Indian National Congress

WWW.INSIGHTSIAS.COM WWW.INSIGHTSONINDIA.COM
INSIGHTSIAS PRELIMS TEST SERIES 2019 – HISTORY QUESTIONS

issued a declaration on the a) 1 and 2 only


Fundamental Rights of Citizenship in
b) 3 only
India. The declaration provided for
c) 1 only
1. Woman shall have the right to vote, to
represent and the right to hold public d) 2 and 3 only
offices.

2. All citizens are equal before the


law, irrespective of religion, caste, creed or Q 140. Consider the following
sex. statements.

Which of the above is/are correct? 1. M.S. Subbulakshmi founded the Indian
Home Rule Society.
a) 1 only
2. Chaitanya Mahaprabhu was an ardent
b) 2 only opponent of the Vaishnava School of Bhakti
Yoga.
c) Both 1 and 2
3. Jagjivan Ram founded the All India
d) None Depressed Classes League.
Select the correct answer using the codes
below.
Q 138. The Indian National
Congress made the demand for a a) 3 only
Constituent Assembly only during/in
b) 1 and 2 only
a) The year 1934
c) 1 and 3 only
b) Quit India movement
d) 1, 2 and 3
c) Non-cooperation movement
d) The Second World War
Q 141. Identify the important
elements in the popular Indian
movements:
Q 139. Consider the following
political parties and their place of 1. Social inequality and the unequal
establishment. distribution of Resources

1. Communist Party of India: West 2. Capturing of Parliamentary


Bengal democracy by the elite

2. Congress Socialist Party: Madras 3. Exploitation of peasants

3. Swaraj Party: Gaya Select the correct answer using the codes
below.
Select the correct answer using the codes
below. a) 1 only

WWW.INSIGHTSIAS.COM WWW.INSIGHTSONINDIA.COM
INSIGHTSIAS PRELIMS TEST SERIES 2019 – HISTORY QUESTIONS

b) 1 and 3 only NATIONAL MOVEMENT 1939-


c) 1, 2 and 3 INDEPENDENCE

d) 2 and 3 only WORLD WAR 2


Q 144. The Muslim League
celebrated the “Deliverance Day” in
Q 142. During the colonial era, the British India in the wake of
“India League” was a Britain-based a) Passage of the Government of India Act
organization whose main aim was to 1935 without including the Muslim League
a) Eradicate untouchability from India members in the drafting committee

b) Increase the representation of Indian MPs b) Congress Ministries resigning from the
in British Parliament Provinces in 1939 on account of the British
involving Indians in the Second World War
c) Work for the welfare of Indian without consulting them
immigrants in Britain
c) Khilafat agitation movement in 1919 on
d) Achieve full independence and self- account of the dethroning of the spiritual
government for India leader of the Islamic world
d) Boycott of the Swadeshi movement as
organized by the Indian National Congress
Q 143. Consider the following about (INC) which did not appeal to the Muslim
the “Red Shirts Movement” in the elite of Calcutta
colonial era.
1. It was a movement to overthrow the
British Empire with massive force and Q 145. With regards to the 1937
violence. provincial elections, consider the
following statements.
2. It was led my Mohammad Ali Jinnah.
1. This was the first time that
3. It eventually turned and transformed into elections to the provincial legislatures were
the Muslim League. held.
Select the correct answer using the codes 2. Majority of population had the
below. right to vote.
a) 1 only 3. The Indian National Congress
(INC) won absolute majority of seats in all
b) 2 and 3 only the provinces.
c) 3 only 4. The All India Muslim League failed to
d) None of the above win even a single seat in the North West
Frontier Province (NWFP).
Select the correct answer using the codes
below.

WWW.INSIGHTSIAS.COM WWW.INSIGHTSONINDIA.COM
INSIGHTSIAS PRELIMS TEST SERIES 2019 – HISTORY QUESTIONS

a) 1, 3 and 4 only b) Establishing a system of provincial


elections and giving them greater autonomy
b) 3 only
c) Constituent Assembly of India elected on
c) 1, 2 and 4 only the basis of adult franchise
d) 1 and 4 only d) Withdrawal of involvement of Indian
troops in the Second World War
QUIT INDIA MOVEMENT
AUGUST OFFER
Q 149. Consider the following
Q 146. The ‘August Offer’ of 1940 statements about the Quit India
accepted which of these important
movement.
demands of the Indian freedom
movement leadership? 1. It was started by Mahatma Gandhi
following a resolution passed in 1942 in
a) Rejection of the idea of partition of India Wardha.
b) Establishing a system of provincial 2. After the movement started, the Congress
elections and giving them greater autonomy was declared an unlawful association,
c) Constituent Assembly of India elected on leaders were arrested and its offices all over
the basis of adult franchise the country were raided and their funds were
frozen.
d) Withdrawal of involvement of Indian
troops in the Second World War 3. Mahatma Gandhi was imprisoned
during the movement.
Select the correct answer using the codes
Q 147. The first Satyagrahi to start below.
Individual Satyagraha on the behest
of Gandhiji was a) 1 only

a) Motilal Nehru b) 1 and 3 only

b) Vinoba Bhave c) 2 and 3 only

c) Sardar Patel d) 1, 2 and 3

d) Acharya Kriplani
OTHER EVENTS
Q 150. The Indian National Army
Q 148. The ‘August Offer’ of 1940 (INA) was created by Netaji Subhash
accepted which of these important Chandra Bose near the
demands of the Indian National
Congress leadership? a) Swadeshi Movement

a) Rejection of the idea of partition of India b) First World War


c) Second World War

WWW.INSIGHTSIAS.COM WWW.INSIGHTSONINDIA.COM
INSIGHTSIAS PRELIMS TEST SERIES 2019 – HISTORY QUESTIONS

d) Civil Disobedience Movement 4. Congress ministries come to power in


some provinces of British India
Select the correct answer using the codes
below.
Q 151. With reference to Azad Hind a) 1243
Government and related
developments, consider the b) 1423
following statements.
c) 4123
1. Netaji Subhash Chandra Bose had
d) 1342
announced the establishment of the
provisional government of Azad Hind in
occupied Singapore in 1943.
CABINET MISSION
2. It was supported by the Axis powers of
Q 153. The Cabinet Mission
Imperial Japan, Nazi Germany, the Italian
included which of these as members?
Social Republic, and their allies.
1. MK Gandhi
3. Netaji Bose was to be the head of the state
of the provisional government, the prime 2. Mountbatten
minister and the minister for war and foreign
affairs. 3. Stafford Cripps

Select the correct answer using the codes Select the correct answer using the codes
below. below.

a) 1 and 2 only a) 2 only

b) 2 and 3 only b) 1 and 3 only

c) 3 only c) 3 only

d) 1, 2 and 3 d) None of the above

Q 152. Arrange the following events TOWARDS INDEPENDENCE AND PARTITION


in chronological order: Q 154. Consider this statement made
in August 1947.
1. The Muslim League moves a resolution at
Lahore demanding a measure of autonomy “Today you have worn on your heads a
for the Muslim-majority areas crown of thorns. The seat of power is a nasty
thing. You have to remain ever wakeful on
2. The British Cabinet sends a three-member that seat….you have to be more humble and
Cabinet Mission to Delhi forbearing…now there will be no end to
3. The Muslim League decides on “Direct your being tested.”
Action” for winning Pakistan The statement was made by

WWW.INSIGHTSIAS.COM WWW.INSIGHTSONINDIA.COM
INSIGHTSIAS PRELIMS TEST SERIES 2019 – HISTORY QUESTIONS

a) M.K Gandhi a) 1 only


b) Jawahar Lal Nehru b) 2 only
c) Maulana Abul Kalam Azad c) Both 1 and 2
d) Mohammed Ali Jinnah d) None

Q 155. With reference to the Q 157. Why 15th August was chosen
Mountbatten Plan, consider the as the Independence Date of India?
following statements.
a) It was on this date that Indian
1. The plan was announced after the Independence Bill was introduced in the
agitation by the Muslim League demanding British House of Commons and passed.
partition of the country.
b) Lord Mountbatten chose the date as he
2. The plan was rejected by the considered this date to be lucky.
Congress and the Muslim League.
c) It was on this date that the Lahore Session
3. Immediate effect was given to the of the Congress passed the “Purna Swaraj”
plan by enacting the Indian Independence resolution.
Act.
d) On this date, India’s Constituent
Select the correct answer using the codes Assembly passed the “Objectives
below. Resolution” granting freedom to India.
a) 1 only
b) 2 and 3 only Q 158. On the 15th August, 1947,
Mahatma Gandhi did not participate
c) 1 and 3 only in any of the Independence Day
d) 2 only celebrations because
1. He was in Hyderabad in the areas which
were torn by gruesome riots between Hindus
Q 156. Consider the following and Muslims.
statements about Partition of India.
2. He did not approve of the Mountbatten
1. Punjab and Bengal were the two plan and the Indian Independence Act 1947.
provinces divided or separated on the basis
Which of the above is/are correct?
of religion.
a) 1 only
2. The scheme of Partition included an
overall plan for transfer of population across b) 2 only
the border.
c) Both 1 and 2
Which of the above is/are correct?
d) None

WWW.INSIGHTSIAS.COM WWW.INSIGHTSONINDIA.COM
INSIGHTSIAS PRELIMS TEST SERIES 2019 – HISTORY QUESTIONS

c) 1 and 2 only

Q 159. C. Rajagopalachari's formula d) 2 and 3 only


(or C. R. formula) was a proposal to
a) Solve the political deadlock between the
All India Muslim League and the Indian CONSTITIONAL CHANGES
National Congress on the independence of Q 161. Consider the following
British India statements.

b) Bring a general acceptance to the 1. Regulating Act of 1773 designated the


introduction of Hindi in the Madras Scheme Governor of Bengal as the ‘Governor-
of Elementary Education General of Bengal’ and created an Executive
Council of four members to assist him.
c) Work towards temple entry proclamations
in the Madras Presidency and worked 2. Charter Act of 1853 separated, for the
towards the upliftment of Dalits first time, the legislative and executive
d) Ensure a balance between the functions of the Governor-General’s
participation of India in the Second World council.
War and totally boycott of the War
3. The Act of 1919 provided, for the first
time, for the association of Indians with the
executive Councils of the Viceroy and
ADMINISTRATIVE CHANGES Governors.
Q 160. Consider the following with
reference to the Indianization of Select the correct answer using the codes
Colonial British administration. below.

1. Following Lord Macaulay’s Report of the a) 1 and 2 only


Select Committee of British Parliament, the
b) 2 and 3 only
concept of a merit based modern Civil
Service in India was introduced in 1854. c) 1 and 3 only
2. It was following the Charter Act of 1891 d) 1 only
that the Indian Civil Service Examination
began to be held in India apart from being
held in London.
Q 162. Montagu's statement made in
3. Entry into Imperial Police was thrown August 1917 promised
open to Indians only with the passing of the
Government of India Act of 1858. 1. Increasing participation of Indians
in every branch of administration
Select the correct answer using the codes
below. 2. Gradual development of institutions with
a view to the progressive realisation of
a) 1 only responsible government
b) 1 and 3 only Which of the above is/are correct?

WWW.INSIGHTSIAS.COM WWW.INSIGHTSONINDIA.COM
INSIGHTSIAS PRELIMS TEST SERIES 2019 – HISTORY QUESTIONS

a) 1 only d) None
b) 2 only
c) Both 1 and 2 Q 165. The first Law Commission
was established, under the
d) None Chairmanship of Lord Macaulay
which recommended codification of
the Penal Code and the Criminal
Q 163. Which of the following was a Procedure Code, by the
salient feature of the Act of 1892?
a) Government of India Act 1935
a) It increased the functions of legislative
b) Charter Act of 1833
councils and gave them the power of
discussing the budget. c) Law Commission Act, 1967
b) It introduced responsible governments in d) Act of 1919
provinces where governor was required to
act with the advice of ministers.
c) It abolished dyarchy in the provinces and GOVERNOR GENERALS AND VICEROYS
established it at the Centre. Q 166. Who came to be known as
the Father of Communal Electorate
d) It introduced bicameralism in many
in India?
eleven provinces.
a) Montague
b) Minto
Q 164. Consider the following
statements about women suffrage c) Cornwallis
in India before independence.
d) Wellesely
1. In 1919 in the Montagu–Chelmsford
Reforms, the British set up provincial
legislatures which had the power to grant
Q 167. Which of these appointed
women's suffrage.
the last Governor General of free
2. Bengal was the first state of British India India?
to grant votes to wealthy and educated
a) Prime Minister of the Interim
women, under the same terms that applied
government
to men.
Which of the above is/are correct? b) Constituent Assembly

a) 1 only c) British Crown

b) 2 only d) Predecessor Viceroy


c) Both 1 and 2

WWW.INSIGHTSIAS.COM WWW.INSIGHTSONINDIA.COM
INSIGHTSIAS PRELIMS TEST SERIES 2019 – HISTORY QUESTIONS

Q 168. Consider the following Cornwallis as the Governor-General


statements. of India?

1. He was the Governor General of Bengal 1. Foundation of the Asiatic Society


when the Permanent Settlement was of Bengal
introduced there in 1793.
2. Permanent Settlement of Bengal
2. He defeated Tipu Sultan in third Anglo- Which of the above is/are correct?
Mysore war and signed Treaty of
Srirangpatanam. a) 1 only

3. He introduced administrative and police b) 2 only


reforms in colonial India.
c) Both 1 and 2
4. He declared the practice of Slavery to be
d) None
illegal.

The above refers to?


Q 171. Which of these authorities
a) Charles Cornwallis
appointed the last Governor General
b) Cole Mckenzie of free India?

c) Warren Hastings a) Prime Minister of the Interim government


b) Constituent Assembly
d) Dalhousie
c) British Crown
d) Predecessor Viceroy
Q 169. Which of the following was a
major factor leading to Warren
Hastings' impeachment in England?
ECONOMIC IMPACT
a) Strained relationships with Chait Singh,
the Maharaja of Benaras Q 172. The Transport of Native
Labourers Act (No. III) of 1863 of
b) Act of 1781, under which the powers of Bengal as amended in 1865, 1870
jurisdiction between the governor-general in and 1873 was mainly associated
council and the Supreme Court at Calcutta, with the forced hiring of labour for
were clearly divided which of the following industries?
c) Third Mysore War (1790-92) and Treaty a) Tea
of Seringa-patam (1792)
b) Coffee
d) Permanent Settlement of Bengal, 1793
c) Cotton
d) Indigo
Q 170. Which of the following
materialized during the tenure of

WWW.INSIGHTSIAS.COM WWW.INSIGHTSONINDIA.COM
INSIGHTSIAS PRELIMS TEST SERIES 2019 – HISTORY QUESTIONS

Q 173. With reference to the 6. Movement from barter trade to


Zamindari system introduced in money economy in India
colonial India, consider the
following statements. Select the correct answer using the codes
below.
1. The profit accruing out of the agriculture
sector went to the cultivators instead of the a) 1, 2, 5 and 6 only
zamindars, but the cultivators were supposed
b) 1, 2, 3, 5 and 6 only
to share the profits with zamindars.
c) 3 and 4 only
2. The Zamindars were officially assigned
the trusteeship of the land under them to d) 1, 2, 3, 4, 5 and 6
improve its condition.
3. The dates for depositing specified sums of
revenue from the zamindari land were fixed, Q 175. With reference to the
failing which the zamindars were to lose industrialization of India induced by
their land rights. the British, consider the following
statements with reference to its
Select the correct answer using the codes impact, as also noted by the Census
below. reports of the 20th CE.
a) 2 and 3 only 1. Establishment of western type
industries gave an impetus to the village
b) 3 only
industries.
c) 1 and 3 only
2. The high prices of agricultural produce,
d) 1 and 2 only owing to industrialization, led many village
artisans to abandon their hereditary craft in
favour of agriculture.
Q 174. Commercialisation of 3. Increase in the pace of
agriculture is a phenomenon industrialization led more people moving
associated with the British Rule in into urban areas.
India. Which of the following factors
increased the pace at which this Select the correct answer using the codes
commercialization occurred? below.

1. American Civil War a) 1, 2 and 3

2. Opening of Suez Canal b) 2 only

3. Replacement of old sailing ships with c) 1 and 3 only


steamships in Britain d) 3 only
4. Depression in British shipping industry
5. Extension of railways in India
Q 176. When the British took over
Indian (colonial) states, some famous

WWW.INSIGHTSIAS.COM WWW.INSIGHTSONINDIA.COM
INSIGHTSIAS PRELIMS TEST SERIES 2019 – HISTORY QUESTIONS

towns lost their courts and thus their d) 1, 2 and 3


artisans and court gentry. These
include
1. Thanjavur Q 178. Over the nineteenth century,
Britain’s trade surplus in India
2. Dhaka helped them pay the ‘home charges’.
This included
3. Murhidabad
1. Private remittances sent home by
Select the correct answer using the codes British officials and traders
below.
2. Interest payments on India’s
a) 1 only external debt
b) 2 and 3 only 3. Pensions of British officials in
India
c) 1 and 3 only
Select the correct answer using the codes
d) 1, 2 and 3
below.
a) 1 only
Q 177. The most important
b) 1 and 3 only
characteristic of India’s foreign trade
throughout the colonial period was c) 1, 2 and 3
the generation of a large export
surplus. However, this was d) 2 and 3 only
problematic because
1. Several essential commodities like food
grains became conspicuous by their acute Q 179. Consider the following
scarcity in the domestic market. statements about the effect of
commercialization of agriculture in
2. It led to an unprecedented flow of gold British India.
and silver into India which devalued the
Indian currency. 1. It brought about a significant and
sustained decline in the prices of agricultural
3. The surplus was used to make payments produce due to the widespread availability
for the expenses incurred by an office set up and ready markets.
by the colonial government in Britain
leading to drain of wealth. 2. Price movements and business
fluctuations in the world markets began to
Select the correct answer using the codes affect the incomes of Indian farmers.
below.
Select the correct answer using the codes
a) 1 and 2 only below.
b) 1 and 3 only a) 1 only
c) 2 and 3 only b) 2 only

WWW.INSIGHTSIAS.COM WWW.INSIGHTSONINDIA.COM
INSIGHTSIAS PRELIMS TEST SERIES 2019 – HISTORY QUESTIONS

c) Both 1 and 2 c) Land holdings were completely


fragmented in the South necessitating
d) None of the above individual clearance of land revenue.
d) Agriculture was still traditional in the
South as compared to the Northern India.
Q 180. Under the Mahalwari
settlement
a) Most villages were made to pay equal Q 183. With reference to the
land revenue irrespective of village economic conditions in India during
population the 1850s-60s, consider the
following statements.
b) Revenue was to be fixed on a permanent
basis 1. The economic conditions in India had
worsened due to a crash in cotton prices
c) The charge of collecting the revenue and
after the end of the American civil war in
paying it to the Company was given to the
1864.
village headman
2. Government's unwillingness to increase
d) Peasants became directly accountable to
land rent and consequent lack of interest in
the Zamindar for payment of land revenue
improving land productivity caused a
succession of bad harvests during this
period.
Q 181. The Book Poverty and
Unbritish Rule in India, which Which of the above is/are correct?
offered a scathing criticism of the a) 1 only
economic impact of British rule was
written by b) 2 only

a) Romesh Chandra Dutt c) Both 1 and 2

b) Lal Bahadur Shashtri d) None

c) M. G. Ranade

d) Dadabhai Naoroji Q 184. In the context of Modern


Indian History, Patuas and Kumors
were

Q 182. Thomas Munro gradually a) Puppeteers and traders


extended the Ryotwari system all
over south India because b) Painters and Potters

a) There were no traditional Zamindars in c) Agricultural labour and royal craftsmen


the South. d) Traders and Iron ore Smelters
b) The land in the South was not as
productive as that of Northern India.

WWW.INSIGHTSIAS.COM WWW.INSIGHTSONINDIA.COM
INSIGHTSIAS PRELIMS TEST SERIES 2019 – HISTORY QUESTIONS

Q 185. Who famously remarked in b) 1 and 3 only


1853: "It was the British intruder
who broke up the Indian handloom c) 1, 2 and 4 only
and destroyed the spinning-wheel.
d) 1, 2, 3 and 4
England began with depriving the
Indian cottons from the European
market; it then introduced twist into
Hindustan and in the end inundated DEVELOPMENT OF PRESS
the very mother country of cotton Q 188. Sankhyā is a quarterly peer-
with cottons”? reviewed scientific journal started by
a) Adam Smith a) Ramesh Chandra Datt
b) Karl Marx b) Prasanta Chandra Mahalanobis
c) James Mill c) Mahadev Goving Ranade
d) Abbot, James d) H.O. Olcott

Q 186. Which of these events gave Q 189. The dissatisfaction with


the first major boost to modern British rule intensified in the 1870s
industries in India in around 1860s? and 1880s. Consider the following
a) Yom Kippur War with reference to acts passed in this
period.
b) French wars waged by Napolean
1. The Arms Act banned domestic
c) American Civil War arm manufacturers and provided for duty
free arm imports.
d) Opium wars
2. The Vernacular Press Act banned
all press agencies in India that were run or
managed by Indians.
Q 187. In colonial India, which
among the following were major Which of the above is/are correct?
centres of textile production?
a) 1 only
1. Chanderi
b) 2 only
2. Dacca
c) Both 1 and 2
3. Kodarma
d) None
4. Banaras
Select the correct answer using the codes
below.
a) 2 and 4 only

WWW.INSIGHTSIAS.COM WWW.INSIGHTSONINDIA.COM
INSIGHTSIAS PRELIMS TEST SERIES 2019 – HISTORY QUESTIONS

DEVELOPMENT OF EDUCATION c) 2 and 3 only


Q 190. The Wood’s Despatch 1854 d) 1 and 3 only
came to India with which of the
following objectives?
1. Deciphering ancient Indian texts to bring Q 192. The Wood’s Despatch 1854
out their relevance to the modern society came to India with which of the
following objectives?
2. To educate the natives of India so that a
class of public servants could be created. 1. Deciphering ancient Indian texts to bring
out their relevance to the modern society
3. To develop practical and vocational skills
of the Indians to increase production in India 2. To educate the natives of India so
that a class of public servants could be
4. Promote education without any
created.
cost imposed on the public in India
3. To develop practical and vocational skills
Select the correct answer using the codes
of the Indians to increase production in India
below.
4. Promote education free of cost in
a) 2 and 3 only
India
b) 1, 3 and 4 only
Select the correct answer using the codes
c) 2 only below.

d) 1, 2, 3 and 4 a) 2 and 3 only


b) 1, 3 and 4 only

Q 191. The education conference c) 2 only


held at Wardha in October 1937 is d) 1, 2, 3 and 4
notable for
1. Promoting free and compulsory education
for an individual throughout his life Q 193. Consider the following
statements about events that
2. Endorsing Gandhiji's proposals for 'basic
transpired during the first decade of
education' through the vernacular medium
the 20th CE.
and manual productive work
1. Bengal National College, inspired by
3. Motivating teachers to work without any
Tagore's Shantiniketan, was set up with
payment for the welfare of nation at large.
Debendranath Tagore as its principal.
Select the correct answer using the codes
2. A National Council of Education was set
below.
up to organize a system of education—
a) 1 and 2 only literary, scientific and technical—on
national lines and under national control.
b) 2 only

WWW.INSIGHTSIAS.COM WWW.INSIGHTSONINDIA.COM
INSIGHTSIAS PRELIMS TEST SERIES 2019 – HISTORY QUESTIONS

3. Swadesh Bandhab Samiti of Ashwini


Kumar Dutta emerged as a popular method
of mass mobilisation. Q 196. Wood’s despatch,
Introduction of the Doctrine of
Select the correct answer using the codes Lapse and opening of
below. Anglovernacular schools and
a) 1 only government colleges is associated
with the tenure of
b) 2 and 3 only
a) Dalhousie
c) 1 and 3 only
b) Dufferin
d) 1, 2 and 3
c) Cornwallis
d) Hastings
Q 194. The Hunter Commission
setup under the tenure of Ripon as
the Governor-General d India (1880- ADMINISTRATIVE POLICY
1884) concerned reforms in Q 197. Which of the following had a
a) Education bearing on the establishment of a
Public Service Commission in India?
b) Factory Act, 1881
1. First Dispatch of the Government of India
c) Local self-government on the Indian Constitutional Reforms, 1919
d) Vernacular Press Act 2. Lee Commission, 1924
3. Whitley Commission, 1929
Q 195. Consider the following 4. Government of India Act, 1935
statements.
Select the correct answer using the codes
1. Indian Universities Act 1904 ensured below.
greater autonomy and relaxed government
a) 1, 2 and 4 only
control over universities considerably.
b) 1, 3 and 4 only
2. Official secret Act 1923 curbed freedom
of press and increased the power of c) 1 and 4 only
magistrates.
d) 2 and 3 only
Which of the above is/are correct?
a) 1 only
Q 198. Consider the following
b) 2 only statements.
c) Both 1 and 2 1. The Lewellyn Smith Committee (1920)
d) None re-examined the tenure system operating
from Lord Curzon's time in 1905.

WWW.INSIGHTSIAS.COM WWW.INSIGHTSONINDIA.COM
INSIGHTSIAS PRELIMS TEST SERIES 2019 – HISTORY QUESTIONS

2. The Maxwell Committee (1937) c) The kingdom evicted the British


suggested that the secretary of the Residents appointed by the Monarch
department should be responsible to the
minister concerned. d) Good governance practices were not
followed by the Kingdom
Which of the above is/are correct?
a) 1 only
Q 201. Tilak's opposition to the Age
b) 2 only of Consent Bill primarily stemmed
from the issue that
c) Both 1 and 2
a) A colonial government should not dictate
d) None such reforms in India
b) The British were morally corrupt
Q 199. Consider the following about c) The bill was ill conceived and
the policy of “paramountcy”. retrogressive in nature
1. East India Company was abolished and d) It bypassed British Parliament and was
India began to be governed directly by the passed as an ordinance
British Monarch
2. The British assured the Indian states that
their territory would not be annexed in the Q 202. Categorisation of Civil
future. services into Imperial, Provisional
and subordinate was done under
Which of the above is/are correct?
the tenure of
a) 1 only
a) Lord Lytton
b) 2 only
b) Lord Curzon
c) Both 1 and 2 c) Lord Lansdowne
d) None d) Lord Dufferin

Q 200. Under Dalhousie’s “Doctrine Q 203. Which of these events,


of Lapse” policy, a kingdom would among the following, in the history
become part of the Company of colonial urban development in
territory if India occurred the last?
a) An Indian ruler died without a male heir a) Dutch establishing their base in
b) The Kingdom did not accept the land Masulipatnam
revenue systems of the British b) First Railway line from Bombay to Thane

WWW.INSIGHTSIAS.COM WWW.INSIGHTSONINDIA.COM
INSIGHTSIAS PRELIMS TEST SERIES 2019 – HISTORY QUESTIONS

c) Beginning of elected representatives in


municipalities
Q 206. The first state of British India to
d) First screening of a film at Watson’s grant votes to wealthy and educated
Hotel, Bombay women, under the same terms that
applied to men was
a) Bengal
Q 204. Hill stations were a
distinctive feature of colonial urban b) United Provinces
development in India. Consider the c) Madras
following about it (assuming the
timelines of the given events to be d) Bombay
correct).
1. Simla (present-day Shimla) was founded
Q 207. Consider the following
during the course of the Gurkha War (1815-
statements.
16).
1. The Religious Disabilities Act, 1856
2. The Anglo-Maratha War of 1818 led to
provided that a change of religion did not
British interest in Mount Abu as a Hill
debar a son from inheriting the property of
station.
his heathen father.
3. Darjeeling was wrested by the British
2. The Caste Disabilities Removal Act, 1850
from the rulers of Sikkim in 1835. abolished all laws affecting the rights of
Select the correct answer using the codes Indian persons converting to another
below. religion or caste.

a) 1 and 2 only Which of the above is/are correct?

b) 2 only a) 1 only

c) 1 and 3 only b) 2 only

d) 1, 2 and 3 c) Both 1 and 2


d) None

Q 205. Which of the following hill


stations served as the official
Q 208. The Cornwallis Code of 1793
residence of the commander-in-chief
provided for
of the Indian army in colonial India?
1. Taking away the police power from the
a) Mussorie
Zamindars
b) Dehradun
2. The restriction of the application of
c) Simla (Shimla) English law to Englishmen only

d) Lansdowne

WWW.INSIGHTSIAS.COM WWW.INSIGHTSONINDIA.COM
INSIGHTSIAS PRELIMS TEST SERIES 2019 – HISTORY QUESTIONS

3. Accommodation of personal laws of THE INDIAN STATES


Hindus and Muslims together in a new Q 210. Consider the following
secular law statements.
4. Streamlining revenue and judicial 1. In the Nagpur session of Congress in
administration in British India 1920, the linguistic principle was recognised
Select the correct answer using the codes as the basis of the reorganisation of the
below. Indian National Congress party.

a) 1, 2 and 4 only 2. A few days after Independence, the


Maharaja of Manipur, signed the Instrument
b) 4 only of Accession with the Indian government on
maintaining the internal autonomy of
c) 2 and 4 only Manipur.
d) 1 and 3 only 3. The Nizam of Hyderabad did not
immediately enter an Instrument of
Accession with India right after
Q 209. Which of the following independence and instead signed a Standstill
statements about town planning in Agreement in 1947.
British India in early and late 19th Select the correct answer using the codes
century is/are correct? below.
1. There was no standardized code a) 1 and 2 only
governing the construction of private
buildings and public roads during this b) 2 and 3 only
period.
c) 1 and 3 only
2. Funds for town improvement were
raised through public lotteries. d) 1 only

3. By the late 19th CE, the


government took initiative of town planning
Q 211. During the suzerainty of the
and development.
British crown, princely States
Select the correct answer using the codes covered what portion of the land area
below. of the British Indian Empire?

a) 1 and 3 only a) one-third

b) 2 and 3 only b) one-tenth

c) 1, 2 and 3 c) one-fifth

d) 1 and 2 only d) two-third

WWW.INSIGHTSIAS.COM WWW.INSIGHTSONINDIA.COM
INSIGHTSIAS PRELIMS TEST SERIES 2019 – HISTORY QUESTIONS

CIVIL REBELLION eventually suppressed by the


government. The region ‘X’ is
CASTE MOVEMENTS
a) Chottanagpur
Q 212. Which of the following can
be categorized as Dalit Movements b) Telangana
or movements for the cause of Dalits
of India? c) Dang region
1. Satnami Movement d) Andaman
2. Adi Dharma Movement
3. Mahar Movement Q 215. With reference to tribal
movement in colonial India, consider
Select the correct answer using the codes the following statements.
below.
1. The Santhals under Sido and Kanhu rose
a) 1 and 2 only up against the British oppressors declaring
b) 1 and 3 only the end of the Company's rule and asserted
themselves independent in 1854.
c) 2 and 3 only
2. The Ahom revolt was organized against
d) 1, 2 and 3 the British attempt to incorporate the
Ahoms' territories in Assam in the
Company's dominion.
TRIBAL REVOLT 3. The Khasi uprising was against the
Q 213. Kukis' Revolt (1917-19; British recruiting Khasis in the imperial
Manipur) was against the armed forces against the wish of the locals.

a) Non-fulfilment of the pledges of the East Select the correct answer using the codes
India Company after the Burmese War below.
b) Occupation of the hilly region by dikus a) 1 and 2 only
c) Hike in farm tax rates for the locals by the b) 2 and 3 only
British
c) 1, 2 and 3
d) British policies of recruiting labour
during the first World War d) 3 only

Q 214. Kol Uprisings by the Kols of Q 216. Consider the following with
‘X’ region led by Buddho Bhagat reference to the Rampa Rebellion of
(1831) against expansion of British 1879.
rule on their lands and transfer of 1. It was a rebellion against the
their lands to outsiders was
British in the hill tracts of Vishakhapatnam.

WWW.INSIGHTSIAS.COM WWW.INSIGHTSONINDIA.COM
INSIGHTSIAS PRELIMS TEST SERIES 2019 – HISTORY QUESTIONS

2. It was led by the Zamindars. c) Malabar


3. Ban of toddy tapping was an important d) Eastern Bengal
factor behind the rebellion.
Select the correct answer using the codes
below. Q 219. Consider the following with
reference to the Rampa Rebellion of
a) 1 only 1879.
b) 2 and 3 only 1. It was a rebellion against the
British in the hill tracts of Vishakhapatnam.
c) 1 and 3 only
2. It was led by the Zamindars.
d) 1, 2 and 3
3. Ban of toddy tapping was an
important factor behind the rebellion.
PEASANT MOVEMENTS Select the correct answer using the codes
below.
Q 217. The causes of Poligars' a) 1 only
Revolt, 1799-1806, could be that
b) 2 and 3 only
1. The British showed little honour to the
customs of the Poligar’s land and denied due c) 1 and 3 only
share of the crops to the peasants.
d) 1, 2 and 3
2. The British not only waged a series of
wars against the Poligars, but deposed and
even executed several of them. Q 220. The Deccan Riots
Which of the above is/are correct? Commission was appointed to

a) 1 only 1. Remodel the police machinery to


tackle increasing riots in the Deccan region
b) 2 only
2. Initiate proceedings against
c) Both 1 and 2 oppressive Zamindars inciting these riots

d) None Which of the above is/are correct?


a) 1 only

Q 218. Pabna Agrarian Leagues, b) 2 only


during the 1870s and 1880s, were
c) Both 1 and 2
formed in the region of
d) None of the above
a) Uttar Pradesh
b) Hyderabad

WWW.INSIGHTSIAS.COM WWW.INSIGHTSONINDIA.COM
INSIGHTSIAS PRELIMS TEST SERIES 2019 – HISTORY QUESTIONS

Q 221. Which of these peasant 2. Oppression of thikadars in charge


movements happened after the 1857 of revenue collection
Sepoy mutiny?
3. Arbitrary dissolution of local
1. Eka movement panchayat
2. Deccan riots Select the correct answer using the codes
below.
3. Indigo revolt
a) 2 only
Select the correct answer using the codes
below. b) 1 and 3 only

a) 1 only c) 1 and 2 only

b) 2 and 3 only d) 1, 2 and 3

c) 3 only
d) 1, 2 and 3 Q 224. Peasant organisations arose
between 1920 and 1940 in India. The
first such organisation was the Bihar
Provincial Kisan Sabha (1929) which
Q 222. With reference to the Kisan was founded by
Sabha Movement, consider the
following statements. a) Sahajanand Saraswati

1. They were organized mainly due to the b) Dada Bhagwan


efforts of the Home Rule activists in Uttar
c) Ram Manohar Lohia
Pradesh.
2. The movement pre-dates First World d) Birendra Sharma
War.
Which of the above is/are correct? Q 225. Consider the following
a) 1 only movements.

b) 2 only 1. Telangana struggle and the


Tebhaga movement were both peasant
c) Both 1 and 2 movements.
d) None 2. Both the movements did not have
any support or backing of a political party.
Which of the above is/are correct?
Q 223. Eka Movement was inspired
from which of the following issues? a) 1 only
1. High land rent b) 2 only
c) Both 1 and 2

WWW.INSIGHTSIAS.COM WWW.INSIGHTSONINDIA.COM
INSIGHTSIAS PRELIMS TEST SERIES 2019 – HISTORY QUESTIONS

d) None Association (TLA) and the All India Trade


Union Congress (AITUC) was formed.
Which of the above is/are correct?
WORKING CLASS MOVEMENT
Q 226. The oldest trade union a) 1 only
federation in India is
b) 2 only
a) Hind Mazdoor Sabha
c) Both 1 and 2
b) All India Trade Union Congress (AITUC)
d) None
c) Indian National Trade Union Congress
Other Movements
(INTUC)
Q 229. Consider the following
d) Bharatiya Mazdoor Sangh (BMS) statements with reference to colonial
India.
1. Vellalars was an urban caste who
Q 227. Consider the following vehemently opposed locals joining the
statements. British administration.
1. The Trade Union Act, 1926 2. Telugu Komatis were a powerful anti-
recognised trade unions as legal commerce pressure group in Southern India
associations. that had considerable influence on grain
trade in Madras.
2. Trade Disputes Act (TDA), 1929 forbade
trade union activity of coercive or purely Which of the above is/are correct?
political nature.
a) 1 only
Which of the above is/are correct?
b) 2 only
a) 1 only
c) Both 1 and 2
b) 2 only
d) None
c) Both 1 and 2
d) None
Q 230. Sannyasi revolt were the
activities of sannyasis and fakirs
Q 228. Consider the following (Hindu and Muslim ascetics,
statements. respectively) against the East India
Company rule in the late 18th
1. The first trade union was established in century in the region of
Madras by B.P. Wadia, a social worker and
member of the Theosophical Society. a) Bengal

2. During the same year (as in S1), Mahatma b) Chittor


Gandhi founded the Textile Labour c) Awadh

WWW.INSIGHTSIAS.COM WWW.INSIGHTSONINDIA.COM
INSIGHTSIAS PRELIMS TEST SERIES 2019 – HISTORY QUESTIONS

d) Odisha (Orissa) Q 233. Dr. B.R. Ambedkar resigned


from the interim cabinet of
independent India was triggered over
an issue related to
PERSONALITIES
Q 231. As per the views of Subhash a) Dominance of centre in the Indian union
Chandra Bose, freedom implies not
only emancipation from political b) Hindu Code Bill
bondage but also c) Constitutional rights of Dalits
1. Equal distribution of wealth d) Extreme socialist bias of Congress
2. Destruction of communalism and
religious intolerance
Q 234. Consider the following
Which of the above is/are correct? statements.
a) 1 only 1. Rajkumari Amrit Kaur was the Minister
b) 2 only for Health in independent India’s first
ministry and a member of constituent
c) Both 1 and 2 assembly.

d) None 2. Acharya Narendra Dev was a


founding President of the Congress Socialist
Party.
Q 232. Rabindranath Tagore's Which of the above is/are correct?
objections to ‘Nationalism’ and
‘Patriotism’ were that a) 1 only

1. A nation is an organised self-interest of a b) 2 only


people and not a voluntary self-expression
c) Both 1 and 2
of individuals as social beings.
2. Racial organization of any nation was d) None
unacceptable because it suppresses diversity
for the expedience of cultural security.
Q 235. Jayaprakash Narayan was
Which of the above is/are correct? associated with which of the
a) 1 only following parties?

b) 2 only 1. Congress Socialist Party

c) Both 1 and 2 2. Socialist Party

d) None 3. Janta Party


Select the correct answer using the codes
below.

WWW.INSIGHTSIAS.COM WWW.INSIGHTSONINDIA.COM
INSIGHTSIAS PRELIMS TEST SERIES 2019 – HISTORY QUESTIONS

a) 1 only b) 2 and 3 only


b) 3 only c) 1 and 3 only
c) 1 and 3 only d) 1, 2 and 3
d) 1, 2 and 3

Q 238. Consider the following about


Dadabhai Naoroji.
Q 236. Consider the following
statements. 1. He shunned joining the legislative
councils due to an appeal made by M.K.
1. He was the first Indian Governor Gandhi.
General of India.
2. He founded the Gyan Prasarak Mandali
2. He founded Swatantra Party. for the education of adults.
3. He was the first recipient of the 3. He wrote the book “The Economic
Bharat Ratna Award. History of India under Early British Rule”.
Select the correct answer using the codes Select the correct answer using the codes
below. below.
a) Rajendra Prasad a) 1 and 2 only
b) C. Rajagopalachari b) 1 and 3 only
c) Satyendra Nath c) 2 only
d) Surendranath Bose d) 2 and 3 only

Q 237. Consider the following Q 239. The first Indian to join the
statements. Indian Civil Service (ICS) in the
British Era was
Sardar Vallabhbhai Patel
a) Satyendranath Tagore
1. Had been a Congress Leader
b) Rashbehari Bose
2. First Home Minister of
independent India c) G.V. Mavalankar
3. Was a member the committee of the d) Kashi Shivanath
Constituent Assembly on Fundamental
Rights
Select the correct answer using the codes Q 240. Consider the following about
below. the role of Dr. B.R. Ambedkar.
a) 1 and 2 only

WWW.INSIGHTSIAS.COM WWW.INSIGHTSONINDIA.COM
INSIGHTSIAS PRELIMS TEST SERIES 2019 – HISTORY QUESTIONS

1. He was the founder of Republican a) Alexander Cook


Party of India and Swatantra Party.
b) William Nathan
2. He was the law minister in the
first cabinet of post- independence c) George Turnour
India.
d) James Prinsep
3. He once acted as the President of
the Bharatiya Jansangh.
Select the correct answer using the codes Q 243. Which of the following
leaders were associated with Indian
below.
National Congress?
a) 1 and 3 only
1. Badruddin Tyabji
b) 2 only
2. S. Subramania lyer
c) 1 and 2 only
3. Narasimha Mehta
d) 1, 2 and 3
4. Romesh Chandra Dutt
Select the correct answer using the codes
Q 241. Some of the major works of below.
Begum Rokeya Sakhawat Hossain
a) 1, 2 and 4 only
include
b) 2 and 3 only
1. Sultana’s Dream
c) 1 and 4 only
2. Abarodhbasini
d) 1, 2, 3 and 4
3. Padmarag
Select the correct answer using the codes
below. Q 244. Consider the following
statements about the views of
a) 1 and 2 only
Subhash Chandra Bose.
b) 2 and 3 only
1. Bose considered industrialization as a
c) 1 and 3 only potent factor in making India strong and
self-sufficient
d) 1, 2 and 3 only
2. Bose did not share Gandhian ideal
of “Ahmisa”.
Q 242. He was the founding editor of Which of the above is/are correct?
the Journal of the Asiatic Society of
Bengal and is best remembered for a) 1 only
deciphering the Kharosthi and
b) 2 only
Brahmi scripts of ancient India. He is

WWW.INSIGHTSIAS.COM WWW.INSIGHTSONINDIA.COM
INSIGHTSIAS PRELIMS TEST SERIES 2019 – HISTORY QUESTIONS

c) Both 1 and 2 c) 1 and 3 only


d) None d) 1, 2 and 3

Q 245. Who was the first Indian Q 248. Consider the following
woman to become the president of statements About Sardar Vallabhai
the Congress? Patel:
a) Sarojini Naidu 1. In 1918, he led a massive “No Tax
Campaign” that urged the farmers not to pay
b) Kalpana Dutt taxes after the British insisted on tax after
the floods in Kaira.
c) Kamaladevi Ghattopadhyaya
2. He was associated with the 1928 “tax-
d) Nellie Sengupta
hike” problem of the farmers in Bardoli
where he helped strike a deal between the
government and farmers’ representatives.
Q 246. Who among the following
was sometimes referred to as 3. Patel was elected as the President of
“Gandhi's conscience keeper”? Indian National Congress in its Lucknow
session (1931) where the party deliberated
a) Motilal Nehru its future path.

b) Maulana Abul Kalam Azad 4. Sardar Patel is credited with uniting the
princely states in pre-independent India to
c) C. Rajagopalachari build the Republic of India.
d) Sardar Vallabhbhai Patel Which are the correct statements?
a) 1, 2, 3 and 4
Q 247. Consider the following about b) Only 1, 2 and 3
Dr. B R Ambedkar.
c) Only 1, 2 and 4
1. He founded the Bahishkrit
Hitakarini Sabha. d) Only 2, 3 and 4

2. He resigned from the first Cabinet


due to differences on the Hindu Code Bill.
Q 249. The first Indian to join the
3. He converted to Buddhism. Indian Civil Service (ICS) in the
British Era was
Select the correct answer using the codes
below. a) Satyendranath Tagore

a) 2 and 3 only b) Rashbehari Bose

b) 1 only c) G.V. Mavalankar

WWW.INSIGHTSIAS.COM WWW.INSIGHTSONINDIA.COM
INSIGHTSIAS PRELIMS TEST SERIES 2019 – HISTORY QUESTIONS

d) Kashi Shivanath on 26 January 1950. These stamps


contain the images of
1. Ashokan wheel spoke of Dharma
Q 250. Consider the following
prominent Indian leaders and the 2. Trumpets and Indian flag
titles given to them during the
3. The Charkha of Gandhi
freedom movement.
4. A majestic Elephant
1. Surendranath Banerjee A.
Indian Burke Select the correct answer using the codes
below.
2. Dadabhai Naoroji B. Lok
Priya a) 1 and 2 only

3. E.V. Ramawamy Naicker C. Socrates b) 2 and 3 only


of South Asia
c) 1 and 4 only
4. Gopinath Bordoloi D. Grand Old Man of d) 2, 3 and 4 only
India
The correct matches for the above are
Q 253. Consider the popular books
a) 1A, 2D, 3C, 4B
published during the Colonial period
b) 1B, 2C, 3D, 4A and their authors.

c) 1D, 2A, 3B, 4C 1. Economic History of India: D.H.


Buchanan
d) 1C, 2B, 3A, 4D
2. India Divided: James Mill
3. The Land Systems of British India:
Powell Baden
MISC Select the correct answer using the codes
Q 251. The Long Walk to Freedom below.
is an autobiography of
a) 1 and 2 only
a) Martin Luther King
b) 2 and 3 only
b) Mazzini
c) 2 only
c) Garibaldi
d) 3 only
d) Nelson Mandela

Q 254. Consider the following about


Q 252. Three stamps were issued in 'Silk Road'.
1950 to mark the first Republic Day

WWW.INSIGHTSIAS.COM WWW.INSIGHTSONINDIA.COM
INSIGHTSIAS PRELIMS TEST SERIES 2019 – HISTORY QUESTIONS

1. The Silk Road was founded by Zin Wudi


by opening up the first route from China to
the West in the 2nd century BC, as a trading Q 256. Consider the following about
expedition. the use of Cowry shells in Ancient
India and regions around.
2. The Silk Road network criss-
crossed Eurasia. 1. Shell money was first introduced in
Southeast Asia.
3. Maritime routes were an important
part of this network. 2. They were given as donation to
monasteries in India, and they could also
4. The term 'Silk Road' was coined in form part of a ritual function in these
15 CE by Jean Baptiste Travernier.
th
monasteries.
Select the correct answer using the codes 3. There is evidence of the exchange of
below. these shells for goods from Maldivian
traders.
a) 1, 2 and 3 only
Select the correct answer using the codes
b) 2, 3 and 4 only
below.
c) 2 and 3 only a) 2 and 3 only
d) 1 and 4 only b) 1 and 2 only
c) 2 only
Q 255. Which of these foods were d) 1 only
introduced in India from outside
during the Medieval age?
1. Potato Q 257. Sevagram is well known in
the history of Indian freedom
2. Chillies struggle because
3. Groundnut 1. It was the place of Gandhiji's ashram and
his residence from 1936 to his death in
4. Maize 1948.
5. Tomato 2. Gandhiji had initially decided to launch
Select the correct answer using the codes the salt satyagraha from Sevagram instead of
below. sabarmati.

a) 1, 2 and 3 only Which of the above is/are correct?

b) 1, 3 and 5 only a) 1 only

c) 2 and 4 only b) 2 only

d) 1, 2, 3, 4 and 5 c) Both 1 and 2


d) None

WWW.INSIGHTSIAS.COM WWW.INSIGHTSONINDIA.COM
INSIGHTSIAS PRELIMS TEST SERIES 2019 – HISTORY QUESTIONS

c) Creation of Bengal Presidency (like


Bombay and Madras)
Q 258. The Northwest Frontier
Province of British India was of d) Transfer of Indian capital from Calcutta
great strategic importance to the to Delhi
British colonials because
1. It was the overland gateway to India
through the Khyber Pass that could be Q 261. Consider the following about
accessed by the invaders. the National Anthem and National
Song of India.
2. The tribes of the North-West Frontier
Province posed a danger to the British 1. The National Anthem of India was first
empire and needed to be tamed through the sung at the Calcutta Session of the Indian
deployment of Khassadars. National Congress.

Which of the above is/are correct? 2. The National Song was first sung at the
1930 Lahore Session of the Indian National
a) 1 only Congress.

b) 2 only Select the correct answer using the codes


below.
c) Both 1 and 2
a) 1 only
d) None
b) 2 only
c) Both 1 and 2
Q 259. In the colonial city of
Madras, who were the dubashes? d) None of the above

a) Indians who could speak two languages –


the local language and English
Q 262. The first Director-General of
b) Intermediaries between Indian society the Archaeological Survey of India
and British (ASI), often called the father of
Indian archaeology, he had a
c) Both (a) and (b) pioneering role in the archaeology of
the Harappan civilization. He was
d) None of the above
a) Max Muller
b) James Prinsep
Q 260. Curzon-Kitchener
controversy led to c) Alexander Cunningham
a) The resignation of Lord Curzon as Viceroy d) Robert Wheeler
b) Re-distribution of political power
between the Central Legislative Assembly
and provincial assemblies

WWW.INSIGHTSIAS.COM WWW.INSIGHTSONINDIA.COM
INSIGHTSIAS PRELIMS TEST SERIES 2019 – HISTORY QUESTIONS

Post-Independence withdraw from the territories of Goa, Diu


and Daman which were under its colonial
Q 263. The Tashkent Agreement
rule since the fourteenth century.
contained the provisions of:
2. At the time of India’s Independence,
1. A no-war pact between India and
Sikkim was not a fully sovereign country
Pakistan
but a ‘protectorate’ of India.
2. Non-interference in each other's
internal affairs Which of the above is/are correct?
a) 1 only
Which of the above is/are correct?
b) 2 only
a) 1 only
b) 2 only c) Both 1 and 2
d) None
c) Both 1 and 2
d) None
Q 266. The Anandpur Sahib
Resolution asserted
Q 264. Consider the following
a) Regional autonomy of the Sikh regions
statements.
b) Banishing the Gurudwara Sahab
1. Under the leadership of Pandit Nehru,
Prabandhak Committee (GSPC) towards a
India convened the Asian Relations
more democratic organization
Conference right after its Independence.
2. India was one of the participants in the c) The futility of a federal structure for India
Bandung Conference that over the years led d) The separation of religion from state
to the establishment of the Non-Alignment
Movement (NAM).
Which of the above is/are correct? Q 267. Consider the following
statements.
a) 1 only
1. V.K.R.V. Rao, the then Indian Minister of
b) 2 only Education, played an important role in the
c) Both 1 and 2 formation of the International Association of
Sanskrit Studies
d) None
2. A. L. Basham and A. K. Narain, eminent
historians of Ancient India, were behind the
setting up of the International Association of
Q 265. Consider the following Buddhist Studies (IABS).
statements.
Which of the above is/are correct?
1. Although the British Empire in India
came to an end in 1947, Portugal refused to a) 1 only

WWW.INSIGHTSIAS.COM WWW.INSIGHTSONINDIA.COM
INSIGHTSIAS PRELIMS TEST SERIES 2019 – HISTORY QUESTIONS

b) 2 only Q 271. Privy Purse was granted to


the princely states due to which of
c) Both 1 and 2 these reasons?
d) None a) Princely states played a significant role in
bringing constitutional reforms in the British
administration
Q 268. People's Union for Civil b) This was a part of the compromise to get
Liberties (PUCL) is a human rights Princely states on board the Constituent
body formed in India in 1976 by the
Assembly
socialist leader
c) Princely states had significant private
a) Jayaprakash Narayan property and their integration was preceded
b) Vinoba Bhave by an assurance that they will be allowed to
retain some property and be given some
c) E.M.S Namboodiripad government allowances

d) Narayan Ganesh Gore d) There was a revolt in the general


populace with the uprooting of the princely
states from the Independent India
Q 269. Who was leading the protests
against the supposedly unfair
election of Indira Gandhi in 1975 Q 272. Consider the following
that led to proclamation of National statements about the Swatantra Party
Emergency? formed in India.

a) Ram Manohar Lohia 1. It was formed after the 1934


Nagpur resolution of the Congress.
b) Jay Prakash Narayan
2. The Swatantra Party was against land
c) C. P. Mazumdar ceilings in agriculture, and opposed
cooperative farming and state trading.
d) A. N. Ray
3. It was critical of the policy of non-
alignment and maintaining friendly relations
with the United States and advocated closer
Q 270. ‘Kamraj plan’ was related to
the reformation of the ranks of which ties with the Soviet Union.
of the following parties? Select the correct answer using the codes
below.
a) Labour party
a) 2 only
b) Indian National Congress
b) 2 and 3 only
c) Bharatiya Jan Sangh
c) 1 and 2 only
d) Congress Socialist Party
d) 1 and 3 only

WWW.INSIGHTSIAS.COM WWW.INSIGHTSONINDIA.COM
INSIGHTSIAS PRELIMS TEST SERIES 2019 – HISTORY QUESTIONS

Q 275. The origin of Communist


Party (Marxist-Leninist) (CPI-ML)
Q 273. Consider the following was a direct and immediate result of
statements about the Indo-China
conflict in 1962. a) Wardha Session of the Indian National
Congress, 1942
1. After the conflict in 1962, full diplomatic
relations were restored between both the b) Mukti Sangharsh, 1955
nations only in 1984.
c) Naxalbari movement, 1967
2. An important cause of the war was a
dispute over the sovereignty of the widely d) None of the above
separated Aksai Chin and North-East
Frontier Agency (NEFA) border regions.
Q 276. Consider the following
Select the correct answer using the codes statements.
below.
1. Bihar Movement was a movement
a) 1 only initiated by students in Bihar in 1974 led by
b) 2 only Jayaprakash Narayan which later turned into
‘Total Revolution’ movement.
c) Both 1 and 2
2. Bihar Movement led to the first non-
d) None Congress party to form a government at the
Centre in India in 1977.
Which of the above is/are correct?
Q 274. The ‘three language formula’
in India is related to which of the a) 1 only
following? b) 2 only
a) Conservation of endangered tribal c) Both 1 and 2
languages in Sixth Schedule areas only
d) None
b) Diplomatic code for encrypted
communication with foreign observers and
visitors
Q 277. The categorization of Indian
c) Education language policy of the states in Parts A, B, C and the like
government was abolished by

d) Translation of ancient scriptures for a) Indian Independence act, 1947


preservation in traditional knolwdge digital b) The Seventh Constitutional Amendment
library (1956)
c) Declaration of India as a republic in the
Preamble of the Constitution
d) Mountbatten’s Plan, 1947

WWW.INSIGHTSIAS.COM WWW.INSIGHTSONINDIA.COM
INSIGHTSIAS PRELIMS TEST SERIES 2019 – HISTORY QUESTIONS

a) Germany

Q 278. Which of these events b) Italy


happened were contemporary to the
c) Spain
partition of India in 1947?
d) Greece
1. Annexation of Goa by India
2. Integration of Hyderabad
(princely state) in India Q 281. Which of these developments
are linked to the Cold War Period?
Select the correct answer using the codes
below. a) Establishment of the UN
a) 1 only b) Creation of nuclear weapons
b) 2 only c) Emergence of Communist China
c) Both 1 and 2 d) All of the above
d) None of the above

Q 282. Arrange the following in


chronological order:

World History 1. Soviet invasion of Afghanistan


Q 279. Arrange the following events 2. Fall of the Berlin Wall
in chronological order:
3. Disintegration of the Soviet Union
1. First World War begins
4. Russian Revolution
2. Treaty of Versailles
Select the correct answer using the codes
3. Proclamation of the Weimar
below.
Republic
Select the correct answer using the codes a) 4132
below. b) 4123
a) 123 c) 4231
b) 213 d) 2341
c) 132
d) 231
Q 283. Consider the following
statements.

Q 280. Which of these European Assertion (A): India was not a


nations did NOT have a colony in the founding member of the League of Nations.
African Continent?

WWW.INSIGHTSIAS.COM WWW.INSIGHTSONINDIA.COM
INSIGHTSIAS PRELIMS TEST SERIES 2019 – HISTORY QUESTIONS

Reason (R): India had signed the languages in various manuscripts


Versailles Treaty after the First World War. found in India?
In the context of the above, which of these is 1. Oriya script
correct?
2. Grantha script
a) A is correct, and R is an appropriate
explanation of A. 3. Bengali script

b) A is correct, but R is not an appropriate 4. Telugu script


explanation of A.
Select the correct answer using the codes
c) A is incorrect, but R is correct. below.

d) Both A and R are incorrect. a) 1 and 4 only


b) 1, 2 and 4 only

Q 284. Consider the following c) 3 only


statements. d) 1, 2, 3 and 4
1. The cold war international order was uni-
polar in nature with United States (US) as
the sole superpower. Q 286. The Mahavamsa is an epic
poem written in the Pali language
2. The US approach to the cold war was
originally set out in the Truman Doctrine related to the history of which of
which advocated disarmament and these South Asian nations?
demilitarization. a) Nepal
Which of the above is/are correct? b) Bhutan
a) 1 only c) Sri Lanka
b) 2 only d) India
c) Both 1 and 2
d) None Q 287. Consider the following
statements.
1. Silpasastra literature describes the group
of the ‘Three Bhangas’ - Abhanga,
ART AND CULTURE Samabhanga, and Atibhanga.
2. The Indian classical dance of
Odissi is characterized by various Bhangas.
LITERATURE
Q 285. Sanskrit language can be Which of the above is/are correct?
found to be written in which of these
a) 1 only

WWW.INSIGHTSIAS.COM WWW.INSIGHTSONINDIA.COM
INSIGHTSIAS PRELIMS TEST SERIES 2019 – HISTORY QUESTIONS

b) 2 only Ardhaparyanka asana and Achamana


Mudra are associated with which of
c) Both 1 and 2 these popular Indian deities?
d) None a) Krishna
b) Shiva
Q 288. Arrange the following major c) Vishnu
textual traditions in chronological
order. d) Brahma

1. Manusmriti in Sanskrit
2. Natyashashtra of Bharata Q 291. Consider the following
statements.
3. Ashtadhyayi of Panini
1. The ‘Dancing Girl’ in tribhanga posture found
4. Works on Astronomy and from Mohenjo-Daro is the earliest bronze
Mathematics by Aryabhata and sculpture discovered till date.
Varahamihira
2. Himachal Pradesh and Kashmir regions
Select the correct answer using the codes produced bronze images of Buddhist deities as
below. well as Hindu gods and goddesses.

a) 1234 3. An important feature of the Gupta and


Vakataka bronze statutes was that they were
b) 3124 enormous in size, unportable and therefore not
vulnerable to theft or smuggling.
c) 2143
Select the correct answer using the codes
d) 4132 below.
a) 1 and 2 only
Q 289. This officer of the East India b) 2 and 3 only
Company deciphered Brahmi and
c) 1 only
Kharosthi, two scripts used in the
earliest inscriptions and coins in d) 1 and 3 only
Indian subcontinent. He was
ARCHITECTURE
a) Ernest Mackay Q 292. Consider the following
b) R. Wheelstone statements.

c) James Prinsep 1. The Kangra fort includes richly


carved temples with idols embossed in their
d) Alfred Marshall walls.

SCULPTURES 2. Dilwara temples are famous for


Q 290. In the context of Ancient their use of marble and intricate marble
Indian sculptures, The carvings.

WWW.INSIGHTSIAS.COM WWW.INSIGHTSONINDIA.COM
INSIGHTSIAS PRELIMS TEST SERIES 2019 – HISTORY QUESTIONS

Which of the above is/are correct? Q 295. Consider the following


statements with reference to the
a) 1 only temple of Brihadeswara at
Thanjavur.
b) 2 only
1. This temple, alongwith , Gangaikonda
c) Both 1 and 2
Cholapuram and Darasuram was built
d) None during the reigns of Rajaraja Chola and
Rajendra Chola.
2. The paintings found at Brihdewara show
Q 293. Temple building activity narrations and aspects related to Lord Shiva
gained momentum during the such as Nataraja.
Vijayanagar rule. Which of these
was/were some of chief Which of the above is/are correct?
characteristics of the Vijayanagara a) 1 only
temple architecture?
b) 2 only
1. Temples were made without mandapams
to accommodate greater number of people in c) Both 1 and 2
the premises.
d) None
2. Horse was the most common animal
found in the pillars of these temples.
Which of the above is/are correct? Q 296. The Dasara Dibba is one of
the most impressive structures in
a) 1 only Hampi. It was used for which of the
b) 2 only following purposes by the King?
1. Watching army marches and war
c) Both 1 and 2
games
d) None
2. Performing sacred Yagnas
Which of the above is/are correct?
Q 294. Which of the following rock a) 1 only
art sites is a UNESCO World
Heritage Site? b) 2 only

a) Pahargarh Caves c) Both 1 and 2

b) Bhimbetka shelters d) None

c) Kalra shelters
Q 297. The cave is carved into the
d) Belum Caves hard monolithic granite rock face of
Barabar hills, flanked to its left by
the smaller Sudama cave. The

WWW.INSIGHTSIAS.COM WWW.INSIGHTSONINDIA.COM
INSIGHTSIAS PRELIMS TEST SERIES 2019 – HISTORY QUESTIONS

ornamentation on the "curved b) 1-A, 2-B, 3-C


architrave" of the cave consists of
carvings of elephants on their way to c) 1-A, 2-C, 3-B
the stupas. It refers to?
d) 1-C, 2-B, 3-A
a) Kanheri Caves
b) Mogalarajapuram Caves
Q 300. In the Eighteenth Century a
d) Udayagiri Caves number of astronomical structures
were built by Jai Singh II in west-
d) Lomus Rishi caves central India. These are today known
by the common name of
Q 298. Pitalkhora, Ellora, Nashik a) Yantra Vedhshala
and Junnar caves are associated with
which religious sect? b) Samarqand Observatory

a) Only Hinduism c) Grahadhyaya

b) Virashaivas d) Jantar Mantar

c) Buddhism Music
Q 301. Daskathia is a form of
d) Only Alvars
a) Ballad singing prevalent in Odisha
b) Puppetry popular in Rajasthan
c) Tribal miniature painting practiced in
Q 299. Consider these different Nilgiris
styles of architecture as a part of
Hindu temple architecture and the d) Folk dance of Manipuri indigenous
temples where they have been used. population

1. Gadag
architecture A. Kasivisvesvara
Temple
Paintings
Q 302. Consider the following
2. Maru-Gurjara statements about the similarities
architecture B. Konark Sun Temple and differences between Nayaka
paintings and Vijayanagar style of
3. Kalinga
architecture C. paintings.
Chennakesava Temple 1. They are usually cited as examples of two
Select the correct match using the codes styles of painting that show remarkable
below. differences even as they belong to the same
region.
a) 1-B, 2-C, 3-A

WWW.INSIGHTSIAS.COM WWW.INSIGHTSONINDIA.COM
INSIGHTSIAS PRELIMS TEST SERIES 2019 – HISTORY QUESTIONS

2. Both the Nayaka and Vijayanagara 2. They developed as an anti-Sanskrit cult to


paintings depict episodes from the promote culture of tribal regions.
Mahabharata and Nayaka paintings also
Which of the above is/are correct?
depicts scenes from Ramayana and Krishna-
leela. a) 1 only
Which of the above is/are correct? b) 2 only
a) 1 only c) Both 1 and 2
b) 2 only d) None
c) Both 1 and 2
d) None
Dance
Q 305. Kutiyattam is a traditional
Q 303. Consider the following about performing artform in the state of
the Deccani School of Painting. a) Tamil Nadu
Assertion (A): It developed as an offshoot of b) Andhra Pradesh
Mughal School of painting.
c) Kerala
Reason (R): Several Mughal painters
migrated to the Deccan during the period of d) Telangana
Aurangzeb and sought patronage there.
In the context of the above, which of these is
correct?
a) A is correct, and R is an appropriate Q 306. Consider the following about
explanation of A. Chhau dance, often see in cultural
news.
b) A is correct, but R is not an appropriate
explanation of A. 1. It is a martial arts dance performed
exclusively by men.
c) A is incorrect, but R is correct.
2. It is a major cultural symbol of the
d) Both A and R are incorrect. Eastern Himalayan Hills.
3. Bhulya and Hos tribes are associated with
this dance form.
Select the correct answer using the codes
Q 304. Consider the following with below.
reference to Central Indian
paintings. a) 1 and 2 only
1. They take inspiration from Indian epics b) 3 only
and religious texts.
c) 1 and 3 only

WWW.INSIGHTSIAS.COM WWW.INSIGHTSONINDIA.COM
INSIGHTSIAS PRELIMS TEST SERIES 2019 – HISTORY QUESTIONS

d) 2 and 3 only Q 310. The Ambubachi Mela


symbolises the fertility cult of a
goddess popular in North-eastern
India. It was recently celebrated in
Q 307. It is the only dance form from
the state of
North-Eastern India that has been
inscribed in the Representative List a) Assam
of the Intangible Cultural Heritage of
Humanity of UNESCO. Performed b) Nagaland
in temples where performers narrate
the lives and deeds of Krishna c) Tripura
through song and dance, it is d) Mizoram
a) Bhaona
b) Chaubola
Q 311. Which of the following
c) Sankirtana describes “Thuni” most
appropriately?
d) Haathras
a) It is an ancient form of martial arts
practiced in Kerala.

Martial art forms b) It is an environmental conservation


technique of the Goddas.
Q 308. Huyen langlon is a traditional
Martial art form of the state of c) It is a harvest festival celebrated in the
North-eastern India.
a) Assam
d) It is the traditional law making body of
b) Meghalaya
the Nagas.
c) Manipur
d) Arunachal Pradesh
Q 312. ‘Dree’ is an agricultural rite
and a fertility festival of the

Festivals and Fairs a) Nyogmas


Q 309. Behdienkhlam is a major b) Lahiris
festival of the people in the Hills of
c) Shyoks
a) Jaintia
d) Apatanis
b) Mizo
b) Khaso
Q 313. Which of the following Indian
d) Garo festivals is on the UNESCO Intangible
Heritage List?
a) Hornbill Festival

WWW.INSIGHTSIAS.COM WWW.INSIGHTSONINDIA.COM
INSIGHTSIAS PRELIMS TEST SERIES 2019 – HISTORY QUESTIONS

b) Kumbh Mela c) 1 and 2 only


c) Bihu Festival d) None of the above
d) Pushkar Mela

Q 316. Consider the following about


the Indian Council for Cultural
Q 314. The Kumbhmela which is the
Relations (ICCR).
largest public gathering in the world
is held at which of the following 1. It was founded in the wake of 1991
places? Economic Reforms waking up to the
imperatives of greater cultural interaction
1. Haridwar with the West.
2. Ujjain 2. It is chaired by the Prime Minister of
India and the Foreign Secretary serves as the
3. Nashik Vice-Chair of the body.
4. Prayagraj
Which of the above is/are correct?
Select the correct answer using the codes
a) 1 only
below.
A. 1, 2, 3 and 4 b) 2 only

B. 1 and 4 only c) Both 1 and 2

C. 1, 2 and 4 only d) None

D. 2 and 3 only

Institution MISC
Q 315. Consider the following about Q 317. Which of the following
the Indian Council for Cultural places in India host important
Relations (ICCR). synagogues religious places of
worship for the Jews, who have been
1. It was founded before India’s recently recognized as a minority in
independence. Maharashtra?
2. It aims to promote cultural exchange with 1. Cochin
other countries and peoples.
2. Pune
3. It funds research in premier cultural
institutions of India. 3. Kolkata

Select the correct answer using the codes Select the correct answer using the codes
below. below.

a) 2 only a) 1, 2 and 3

b) 1 and 3 only b) 2 and 3 only

WWW.INSIGHTSIAS.COM WWW.INSIGHTSONINDIA.COM
INSIGHTSIAS PRELIMS TEST SERIES 2019 – HISTORY QUESTIONS

c) 1 only b) Cave paintings of Sonbhadra and


Chitrakoot
d) 2 only
c) Fabrics like chiffon, muslin and organza
d) Metalwork of Badohi and decorated
Q 318. Located in Gujarat’s Patan, terracotta horse
the 900-year-old structure, Rani-ki-
Vav, is a major tourist attraction, a Q 321. The logo of ‘International
UNESCO World Heritage site, and Yoga Day’ includes which of these
was awarded as the cleanest iconic elements?
place in India in 2016. It is located
on the banks of the mythical 1. Earth

a) Saraswati River 2. Leaves


3. Elephant
b) Vaitarni River
4. Sun
c) Triveni River
Select the correct answer using the codes
d) Dungamatra River below.
a) 1, 2 and 4 only
Q 319. Homeopathy is a medical b) 2 and 3 only
system based on the belief that
c) 1 and 4 only
1. The body can cure itself
d) 1, 2, 3 and 4
2. A substance that causes the symptoms of
a disease in healthy people would cure
similar symptoms in sick people Q 322. Consider the following
Which of the above is/are correct? statements about the historical
inscriptions in India.
a) 1 only
1. All the edicts of Asoka engraved in the
b) 2 only Kharoshthi and Brahmi scripts are in the
c) Both 1 and 2 Prakrit language.

d) None 2. With the rise of the Guptas,


Prakrit became the predominant language
of Indian epigraphs.
Q 320. Anantnag, Baramula, and
Which of the above is/are correct?
Pahalgam are well known for
a) 1 only
a) Production of Kashmiri willow Cricket
Bats b) 2 only

c) Both 1 and 2

WWW.INSIGHTSIAS.COM WWW.INSIGHTSONINDIA.COM
INSIGHTSIAS PRELIMS TEST SERIES 2019 – HISTORY QUESTIONS

d) None Q 326. Kalam (Kalamezhuthu) is a


unique form of art found in Kerala. It
is essentially a/an
Q 323. In the Indian tradition, a year is a) ritualistic art practiced in temples and
divided into six two-monthly seasons. sacred groves of Kerala
Which of the following months in this
system would correspond to the Winter b) art form with block prints made on
season (Hemanta and Shishira)? bamboo
a) Margashirsha to Phalguna c) technique of embroidery employing
traditions mural paintings
b) Chaitra to Asadha
c) Sravana to Kartik d) None of the above

d) Bhadra to Asvina

Q 327. Consider the following


statements about the inscriptions
Q 324. The Tamil “Tirukkural” found at Aihole, Karnataka.
written by Thiruvalluvar mainly
deals with 1. They are written in a single
language and script.
a) Ethics and way of life
2. They mention poet Kalidasa.
b) Genealogy of major Tamil Kingdoms
3. They do not entertain any matter
c) Politics and administration other than the praise of Kings.
d) Astronomy and Astrology Select the correct answer using the codes
below.
a) 1 and 2 only
Q 325. Kaniyan koothu is a
b) 3 only
a) Ritual art form practised during temple
festivals in Tamil Nadu c) 2 only
b) Bull sports practiced in Rayalseema d) 1, 2 and 3
region
c) Sculpture arts popular among the Nilgiri
tribes
Q 328. Wahhabism is the dominant
d) Harvest song of the primitive tribes of faith of which of the following
Kerala countries?
a) Afghanistan
b) Iraq

WWW.INSIGHTSIAS.COM WWW.INSIGHTSONINDIA.COM
INSIGHTSIAS PRELIMS TEST SERIES 2019 – HISTORY QUESTIONS

c) Saudi Arabia The site corresponding to the above is?


d) Syria a) Moidam
b) Burzahom

Q 329. Iconic Saree Weaving c) Narcondam


Clusters of India are part of The
Tentative Lists of States Parties are d) Lamjano
published by the World Heritage
Centre, UNESCO. Match the
following clusters with the state they
are located in:
1. Chanderi: Madhya Pradesh
2. Paithan: Gujarat
3. Yeola: Rajasthan
4. Pochampalli: Telangana
Select the correct answer using the codes
below.
a) 1 and 2 only
b) 2 and 3 only
c) 1 and 4 only
d) 3 and 4 only

Q 330. Consider the following


statements.
1. It was the Mound-Burial system of the
Ahom Dynasty.
2. The structural construction and the
process of royal burials are explained in
historical documents called Chang-Rung
Phukanor Buranji.
3. The property and Buffer zones are jointly
protected and managed jointly by the
Archaeological Survey of India and the State
Department of Archaeology.

WWW.INSIGHTSIAS.COM WWW.INSIGHTSONINDIA.COM

Das könnte Ihnen auch gefallen